Re: [R] Q. regarding optim()

2006-07-21 Thread Prof Brian Ripley
On Fri, 21 Jul 2006, N. Goodacre wrote:

Dear R mailing group,
 
   The second parameter for the function optim()is a function whose 
 parameters are to be optimized. The description of this function given in 
 the help file is the following:
 
   fn: A function to be minimized (or maximized), with first
   argument the vector of parameters over which minimization is
   to take place.  It should return a scalar result.
 
 Let's say the second argument is x, a vector of x values

But it says `parameters'.  Please look at the examples on that help page.  
optim is concerned with optimizing functions of more than one parameter. 
In a statistical setting 'x' may be the data, but e.g. for fitting a gamma 
distribution c(scale, shape) are the parameters.

 I would have thought that fn should return a vector full of y values for 
 the x values entered as the second argument. If the function just takes one 
 value at a time and outputs a scalar, how can I specify for fn which x 
 value, of the vector x, to take?
 
  Sincerely,
 
 Norman Goodacre
 
 __
 R-help@stat.math.ethz.ch mailing list
 https://stat.ethz.ch/mailman/listinfo/r-help
 PLEASE do read the posting guide http://www.R-project.org/posting-guide.html
 and provide commented, minimal, self-contained, reproducible code.
 

-- 
Brian D. Ripley,  [EMAIL PROTECTED]
Professor of Applied Statistics,  http://www.stats.ox.ac.uk/~ripley/
University of Oxford, Tel:  +44 1865 272861 (self)
1 South Parks Road, +44 1865 272866 (PA)
Oxford OX1 3TG, UKFax:  +44 1865 272595

__
R-help@stat.math.ethz.ch mailing list
https://stat.ethz.ch/mailman/listinfo/r-help
PLEASE do read the posting guide http://www.R-project.org/posting-guide.html
and provide commented, minimal, self-contained, reproducible code.


Re: [R] Parameterization puzzle

2006-07-21 Thread Prof Brian Ripley
R does not know that poly(age,2) and poly(age,1) are linearly dependent.
(And indeed they only are for some functions 'poly'.)

I cannot reproduce your example ('l' is missing), but perhaps

glm(deaths ~ poly(age,2) + poly(age,1)*Smoke + offset(l), poisson)

was your intention?

On Fri, 21 Jul 2006, Murray Jorgensen wrote:

 Consider the following example (based on an example in Pat Altham's GLM 
 notes)
 
 pyears - scan()
 18793 52407 10673 43248 5710 28612 2585 12663 1462 5317
 
 deaths - scan()
 2 32 12 104 28 206 28 186 31 102
 
 Smoke - gl(2,1,10,labels=c(No,Yes))
 Age - gl(5,2,10,labels=c(35-44,45-54,55-64,65-74,75-84),
 ordered=TRUE)
 mod1.glm - glm(deaths ~ Age * Smoke + offset(l),family=poisson)
 summary(mod1.glm)
 age - as.numeric(Age)
 mod2.glm - aso1.glm - glm(deaths ~ poly(age,2) + Smoke +
poly(age,1):Smoke + offset(l),family=poisson)
 summary(mod2.glm)
 
 
 
 The business part of the summary for the first model
 
 Estimate Std. Error z value Pr(|z|)
 (Intercept)-5.927060.16577 -35.754   2e-16 ***
 Age.L   4.064900.47414   8.573   2e-16 ***
 Age.Q  -1.082930.41326  -2.620 0.008781 **
 Age.C   0.241580.31756   0.761 0.446816
 Age^4   0.042440.23061   0.184 0.853986
 SmokeYes0.619160.17296   3.580 0.000344 ***
 Age.L:SmokeYes -1.312340.49267  -2.664 0.007729 **
 Age.Q:SmokeYes  0.390430.43008   0.908 0.363976
 Age.C:SmokeYes -0.295930.33309  -0.888 0.374298
 Age^4:SmokeYes -0.036820.24432  -0.151 0.880218
 
 inspires me to fit the second model that omits the nonsignificant terms, 
 however this produces the summary
 
Estimate Std. Error z value Pr(|z|)
 (Intercept)-5.8368 0.1213 -48.103   2e-16 ***
 poly(age, 2)1   3.9483 0.1755  22.497   2e-16 ***
 poly(age, 2)2  -1.0460 0.1448  -7.223 5.08e-13 ***
 SmokeYes0.5183 0.1262   4.106 4.02e-05 ***
 SmokeNo:poly(age, 1)1.3755 0.4340   3.169  0.00153 **
 SmokeYes:poly(age, 1)   NA NA  NA   NA
 
 Why do we get a SmokeNo:poly(age, 1) term? Can I re-express mod2.glm so 
 that this term does not appear?
 
 Cheers,  Murray Jorgensen
 
 

-- 
Brian D. Ripley,  [EMAIL PROTECTED]
Professor of Applied Statistics,  http://www.stats.ox.ac.uk/~ripley/
University of Oxford, Tel:  +44 1865 272861 (self)
1 South Parks Road, +44 1865 272866 (PA)
Oxford OX1 3TG, UKFax:  +44 1865 272595

__
R-help@stat.math.ethz.ch mailing list
https://stat.ethz.ch/mailman/listinfo/r-help
PLEASE do read the posting guide http://www.R-project.org/posting-guide.html
and provide commented, minimal, self-contained, reproducible code.


[R] insert insertRow?

2006-07-21 Thread Marjo en Edwin
Dear all,

In the search for a command to insert a row between other rows in a data 
frame I found that there seems to be no such command in the base R 
package. There is however a very simple function insertRow in the 
micEcon package, that makes use of rbind. I wondered if it would not be 
possible to include the following micEcon functions in the base package:

insertRow
insertCol

Since the functions are already there, I would gather this is not a very 
big effort. It would save people that just want to insert rows and 
columns easily to install another two packages (since micEcon needs 
systemfit) or defining the functions themselves.

I hope my suggestion will be taken into consideration. Whether it will 
be adopted or not, I still think R is a fantastic statistical package 
that I love to use.

Greetings,
Edwin Commandeur

__
R-help@stat.math.ethz.ch mailing list
https://stat.ethz.ch/mailman/listinfo/r-help
PLEASE do read the posting guide http://www.R-project.org/posting-guide.html
and provide commented, minimal, self-contained, reproducible code.


[R] Merge two dataframes of different column length and row length by two columns at a time

2006-07-21 Thread Gunther Höning

Hello,

I have two dataframes of the following structures:

str(a)
`data.frame':   1354896 obs. of  14 variables:
 $ V1 : int  0 1 2 3 4 5 6 7 8 9 ...
 $ V2 : int  0 0 0 0 0 0 0 0 0 0 ...
 $ V3 : int  74 12305 103 12337 46 57 12446 90 12097 79 ...
 $ V4 : num11.8 1529.2   17.8 1579.46.7 ...
 $ V5 : int  88 11040 104 11557 56 58 11040 74 10991 81 ...
 $ V6 : num15.5 1921.3   20.3 1888.2   12.6 ...
 $ V7 : int  87 8793 90 10142 67 64 9264 73 8589 71 ...
 $ V8 : num16.0 1750.6   15.2 1783.7   11.0 ...
 $ V9 : int  68 11279 93 11831 43 61 11234 82 10919 76 ...
 $ V10: num11.5 1999.5   39.0 1842.25.0 ...
 $ V11: int  110 12456 92 12063 60 59 12393 82 11831 77 ...
 $ V12: num21.4 1974.7   33.9 1689.9   10.6 ...
 $ V13: int  81 10887 101 10874 62 74 5 79 10789 93 ...
 $ V14: num19.5 1812.3   31.7 1524.1   11.9 ...
 str(b)
`data.frame':   1213901 obs. of  4 variables:
 $ V1: int  0 1 2 3 5 6 7 8 9 10 ...
 $ V2: int  0 0 0 0 0 0 0 0 0 0 ...
 $ V3: Factor w/ 54676 levels str23,str53,..: 54676 54676 54676 54676
54676 54676 54676 54676 54676 54676 ...
 $ V4: Factor w/ 3 levels Match,NoMatch,..: 2 2 2 2 2 2 2 2 2 2 ...
 

I want to merge these dataframes by V1 and V2 of a and b. The combination of
V1, V2 is a unique key.
Note that b is smaller than a.

Any suggestions to solve this problem ?





Gunther Höning

Diplom Physiker
Bioinformatiker

Langenbeckstraße1
55131 Mainz

[EMAIL PROTECTED]

__
R-help@stat.math.ethz.ch mailing list
https://stat.ethz.ch/mailman/listinfo/r-help
PLEASE do read the posting guide http://www.R-project.org/posting-guide.html
and provide commented, minimal, self-contained, reproducible code.


Re: [R] Correspondence analysis with R

2006-07-21 Thread Gavin Simpson
On Thu, 2006-07-20 at 14:38 +, Emanuele Mazzola wrote:
 Hello everybody,
 
 i'm having some trouble performing correspondence analysis with R for Mac OS 
 X. Does anyone know about some useful package?
 And also, if i had found coordinates of points representing row and column 
 profiles, how do i put them in a single figure with labels identifying each 
 one of them?
 This thing is getting me almost crazy...
 
 Thank you in advance for answering,
 bye
 Emanuele

A summary of packages/functions providing ordination methods including
CA is in the Environmetrics Task View on CRAN, e.g.:

http://www.stats.bris.ac.uk/R/src/contrib/Views/Environmetrics.html

HTH

G
-- 
%~%~%~%~%~%~%~%~%~%~%~%~%~%~%~%~%~%~%~%~%~%~%~%~%~%~%~%~%~%~%~%~%~%~%~%
 *Note new Address and Fax and Telephone numbers from 10th April 2006*
%~%~%~%~%~%~%~%~%~%~%~%~%~%~%~%~%~%~%~%~%~%~%~%~%~%~%~%~%~%~%~%~%~%~%~%
Gavin Simpson [t] +44 (0)20 7679 0522
ECRC  [f] +44 (0)20 7679 0565
UCL Department of Geography
Pearson Building  [e] gavin.simpsonATNOSPAMucl.ac.uk
Gower Street
London, UK[w] http://www.ucl.ac.uk/~ucfagls/cv/
WC1E 6BT  [w] http://www.ucl.ac.uk/~ucfagls/
%~%~%~%~%~%~%~%~%~%~%~%~%~%~%~%~%~%~%~%~%~%~%~%~%~%~%~%~%~%~%~%~%~%~%~%

__
R-help@stat.math.ethz.ch mailing list
https://stat.ethz.ch/mailman/listinfo/r-help
PLEASE do read the posting guide http://www.R-project.org/posting-guide.html
and provide commented, minimal, self-contained, reproducible code.


Re: [R] Loss of numerical precision from conversion to list ?

2006-07-21 Thread Peter Dalgaard
Duncan Murdoch [EMAIL PROTECTED] writes:

 R tries to use the maximum precision (64 bit mantissa) in the floating 
...
 Or perhaps your problem has nothing to do with this; I didn't really 
 look at it in detail.

It hasn't. I was off speculating about sum vs rowSums too, but:

  num.v-  rowSums(((lambda-lambda0)*mu*w.k.sq[,-(K+1)])/(1+lambda*mu))

Inside this, we have mu*w.k.sq[,-(K+1)] . mu is a vector of length 27,
and w.k.sq has 10 rows and 28 *columns*. Column-major storage and
vector recycling kicks in... If mu has identical elements (never mind
the magnitude), of course, the recycling doesn't matter.

-- 
   O__   Peter Dalgaard Øster Farimagsgade 5, Entr.B
  c/ /'_ --- Dept. of Biostatistics PO Box 2099, 1014 Cph. K
 (*) \(*) -- University of Copenhagen   Denmark  Ph:  (+45) 35327918
~~ - ([EMAIL PROTECTED])  FAX: (+45) 35327907

__
R-help@stat.math.ethz.ch mailing list
https://stat.ethz.ch/mailman/listinfo/r-help
PLEASE do read the posting guide http://www.R-project.org/posting-guide.html
and provide commented, minimal, self-contained, reproducible code.


Re: [R] Correspondence analysis with R -follow up

2006-07-21 Thread Gavin Simpson
On Thu, 2006-07-20 at 16:40 +, Emanuele Mazzola wrote:
 Hi all,
 
 thank you for your answers; i've tried both cca from vegan library, and 
 dudi.coa from ade4 library; one last question: my deal is mainly with 
 contingency tables, like the one i'm posting here
 
 acciaieria-c(.41,.02,.44,.04,.09)
 laminatoio-c(.34,.28,.26,.01,.11)
 fonderia-c(.48,.05,.34,.08,.05)
 leghe-c(.45,.19,.25,.03,.08)
 pct-cbind(acciaieria,laminatoio,fonderia,leghe)
 pct-data.frame(pct,row.names=c(normale,preparaz.,manutenz.,installaz.,trasferim.))
 
 BUT...cca and dudi.coa seem to return quite different graphical results; 
 where am i doing wrong?
 Do they do the same to you with the table above?
 
 Thank you very much again!
 Bye
 Emanuele

Hi, I haven't used ade4 at all, but as long as you did CA correctly
using ade4 functions, I doubt the plotted results differ really in all
but cosmetic ways or perhaps in terms of the scalings used.

You are plotting two bits of information in the biplot and you can only
represent one of those optimally, or you could compromise and plot with
symmetric scaling. Or you could plot them in a multitude of ways - there
are whole books on biplots!

Take a look at argument scaling in ?plot.cca (for vegan). Try plotting
your CA with different scalings and see if that better matches the CA
from ade4, eg:

library(vegan)
mod - cca(pct)
plot(mod) # default is scaling = 2
plot(mod, scaling = 1)
plot(mod, scaling = 3)

If using vegan, you might want to look at Jari Oksanens Vegan Tutorial
for more info on using his functions:

http://cc.oulu.fi/~jarioksa/opetus/metodi/vegantutor.pdf

If this doesn't help your understanding or problem, post back with the
ade4 and vegan code you are using and I'll have a look.

G
-- 
%~%~%~%~%~%~%~%~%~%~%~%~%~%~%~%~%~%~%~%~%~%~%~%~%~%~%~%~%~%~%~%~%~%~%~%
 *Note new Address and Fax and Telephone numbers from 10th April 2006*
%~%~%~%~%~%~%~%~%~%~%~%~%~%~%~%~%~%~%~%~%~%~%~%~%~%~%~%~%~%~%~%~%~%~%~%
Gavin Simpson [t] +44 (0)20 7679 0522
ECRC  [f] +44 (0)20 7679 0565
UCL Department of Geography
Pearson Building  [e] gavin.simpsonATNOSPAMucl.ac.uk
Gower Street
London, UK[w] http://www.ucl.ac.uk/~ucfagls/cv/
WC1E 6BT  [w] http://www.ucl.ac.uk/~ucfagls/
%~%~%~%~%~%~%~%~%~%~%~%~%~%~%~%~%~%~%~%~%~%~%~%~%~%~%~%~%~%~%~%~%~%~%~%

__
R-help@stat.math.ethz.ch mailing list
https://stat.ethz.ch/mailman/listinfo/r-help
PLEASE do read the posting guide http://www.R-project.org/posting-guide.html
and provide commented, minimal, self-contained, reproducible code.


[R] failed installing rgl

2006-07-21 Thread James Foadi
Dear all,
I have tried installing rgl with the usual command:

R CMD INSTALL rgl_0.67-2.tar.gz

Differently from what happened last time I have succesfully installed this 
package, this time there was a failure:

...
...g++ -I/usr/lib/R/include -I/usr/lib/R/include -I -DHAVE_PNG_H 
-I/usr/include/libpng12 -I/usr/local/include   -fpic  -O2 -g -pipe -Wall 
-Wp,-D_FORTIFY_SOURCE=2 -fexceptions -fstack-protector 
--param=ssp-buffer-size=4 -m32 -march=i386 -mtune=generic 
-fasynchronous-unwind-tables -c api.cpp -o api.o
In file included from glgui.hpp:9,
 from gui.hpp:11,
 from rglview.h:10,
 from Device.hpp:11,
 from DeviceManager.hpp:9,
 from api.cpp:14:
opengl.hpp:23:20: error: GL/glu.h: No such file or directory
Disposable.hpp:13: warning: ‘struct IDisposeListener’ has virtual functions 
but non-virtual destructor
types.h:77: warning: ‘class DestroyHandler’ has virtual functions but 
non-virtual destructor
gui.hpp:56: warning: ‘class gui::WindowImpl’ has virtual functions but 
non-virtual destructor
gui.hpp:90: warning: ‘class gui::GUIFactory’ has virtual functions but 
non-virtual destructor
pixmap.h:39: warning: ‘class PixmapFormat’ has virtual functions but 
non-virtual destructor
api.cpp: In function ‘void rgl_user2window(int*, int*, double*, double*, 
double*, double*, int*)’:
api.cpp:707: error: ‘gluProject’ was not declared in this scope
api.cpp: In function ‘void rgl_window2user(int*, int*, double*, double*, 
double*, double*, int*)’:
api.cpp:735: error: ‘gluUnProject’ was not declared in this scope
make: *** [api.o] Error 1
chmod: cannot access `/usr/lib/R/library/rgl/libs/*': No such file or 
directory
ERROR: compilation failed for package 'rgl'
** Removing '/usr/lib/R/library/rgl'
...
...

It is clear that glu.h cannot be found during installation and, indeed, I have 
checked and there is no glu.h under /usr/include/GL.

Any suggestion on how I could proceed?
I am using FEDORA CORE 5.
By typing glxinfo it seems that glu 1.3 is installed. But where's glu.h, then?

Many thanks

J

-- 
Dr James Foadi
Department of Physics
University of York
York YO10 5DD

email: [EMAIL PROTECTED]
Tel: 0044 1904 434622
Mobile: 0044 7740 678548


___ 
All New Yahoo! Mail � Tired of [EMAIL PROTECTED]@! come-ons? Let our SpamGuard 
protect you. http://uk.docs.yahoo.com/nowyoucan.html

__
R-help@stat.math.ethz.ch mailing list
https://stat.ethz.ch/mailman/listinfo/r-help
PLEASE do read the posting guide http://www.R-project.org/posting-guide.html
and provide commented, minimal, self-contained, reproducible code.


Re: [R] Parameterization puzzle

2006-07-21 Thread Berwin A Turlach
G'day all,

 BDR == Prof Brian Ripley [EMAIL PROTECTED] writes:

BDR R does not know that poly(age,2) and poly(age,1) are linearly
BDR dependent.
Indeed, I also thought that this is the reason of the problem.

BDR (And indeed they only are for some functions 'poly'.)
I am surprised about this.  Should probably read the help page of
'poly' once more and more carefully.

BDR I cannot reproduce your example ('l' is missing), [...]
My guess is that 'l' is 'pyears'.  At least, I worked under that
assumption.  

Interestingly, on my machine (using R 2.3.1, 2.2.1 and 2.1.1) I cannot
fit any of the Poisson GLM that Murray tried.  I always get the error
message:

Error: no valid set of coefficients has been found: please supply starting 
values

But I have to investigate this further.  I can fit binomial models
that give me similar answers.

BDR [...] but perhaps
BDR glm(deaths ~ poly(age,2) + poly(age,1)*Smoke + offset(l),
BDR poisson)
BDR was your intention?
In this parameterisation a 'poly(age,1)' term will appear among the
coefficients with an estimated value of NA since it is aliased with
'poly(age, 2)1'.  So I don't believe that this was Murray's intention.

The only suggestion I can come up with is:

 summary(glm(cbind(deaths, l-deaths) ~ age*Smoke+I(age^2), family=binomial))

[...]

Coefficients:
  Estimate Std. Error z value Pr(|z|)
(Intercept)  -10.798950.45149 -23.918   2e-16 ***
age2.378920.20877  11.395   2e-16 ***
SmokeYes   1.445730.37347   3.871 0.000108 ***
I(age^2)  -0.197060.02749  -7.168  7.6e-13 ***
age:SmokeYes  -0.308500.09756  -3.162 0.001566 ** 

[...]

Which doesn't use orthogonal polynomials anymore.  But I don't see how
you can fit the model that Murray want to fit using orthogonal
polynomials given the way R's model language operates.

So I guess the Poisson GLM that Murray wants to fit is:

glm(deaths~ age*Smoke+I(age^2)+offset(l), family=poisson)

Cheers,

Berwin

== Full address 
Berwin A Turlach  Tel.: +61 (8) 6488 3338 (secr)   
School of Mathematics and Statistics+61 (8) 6488 3383 (self)  
The University of Western Australia   FAX : +61 (8) 6488 1028
35 Stirling Highway   
Crawley WA 6009e-mail: [EMAIL PROTECTED]
Australiahttp://www.maths.uwa.edu.au/~berwin

__
R-help@stat.math.ethz.ch mailing list
https://stat.ethz.ch/mailman/listinfo/r-help
PLEASE do read the posting guide http://www.R-project.org/posting-guide.html
and provide commented, minimal, self-contained, reproducible code.


Re: [R] Merge two dataframes of different column length and row length by two columns at a time

2006-07-21 Thread Philipp Pagel
On Fri, Jul 21, 2006 at 09:33:56AM +0200, Gunther Höning wrote:
 I have two dataframes of the following structures:
[...]

 I want to merge these dataframes by V1 and V2 of a and b. The combination of
 V1, V2 is a unique key.
 Note that b is smaller than a.
 
 Any suggestions to solve this problem ?

Use merge()

cu
Philipp

-- 
Dr. Philipp PagelTel.  +49-8161-71 2131
Dept. of Genome Oriented Bioinformatics  Fax.  +49-8161-71 2186
Technical University of Munich
Science Center Weihenstephan
85350 Freising, Germany

 and

Institute for Bioinformatics / MIPS  Tel.  +49-89-3187 3675
GSF - National Research Center   Fax.  +49-89-3187 3585
  for Environment and Health
Ingolstädter Landstrasse 1
85764 Neuherberg, Germany
http://mips.gsf.de/staff/pagel

__
R-help@stat.math.ethz.ch mailing list
https://stat.ethz.ch/mailman/listinfo/r-help
PLEASE do read the posting guide http://www.R-project.org/posting-guide.html
and provide commented, minimal, self-contained, reproducible code.


Re: [R] Parameterization puzzle

2006-07-21 Thread Prof Brian D Ripley
On Fri, 21 Jul 2006, Berwin A Turlach wrote:

 G'day all,

 BDR == Prof Brian Ripley [EMAIL PROTECTED] writes:

BDR R does not know that poly(age,2) and poly(age,1) are linearly
BDR dependent.
 Indeed, I also thought that this is the reason of the problem.

BDR (And indeed they only are for some functions 'poly'.)
 I am surprised about this.  Should probably read the help page of
 'poly' once more and more carefully.

My point was perhaps simpler: if you or I or Murray had a function poly() 
in our workspace, that one would be found, I think.  (I have not checked 
the ramifications of namespaces here, but I believe that would be the 
intention, that formulae are evaluated in their defining environment.)  So 
omly when the model matrix is set up could the linear dependence be known 
(and there is nothing in the system poly() to tell model.matrix).

What is sometimes called extrinsic aliasing is left to the fitting 
function, which seems to be to do a sensible job provided the main effect 
is in the model.  Indeed, including interactions without main effects (as 
Murray did) often makes the results hard to interpret.

BDR I cannot reproduce your example ('l' is missing), [...]
 My guess is that 'l' is 'pyears'.  At least, I worked under that
 assumption.

Apparently l = log(pyears), which was my uncertain guess.

 Interestingly, on my machine (using R 2.3.1, 2.2.1 and 2.1.1) I cannot
 fit any of the Poisson GLM that Murray tried.  I always get the error
 message:

 Error: no valid set of coefficients has been found: please supply starting 
 values

Related to the offset, I believe.


 But I have to investigate this further.  I can fit binomial models
 that give me similar answers.

BDR [...] but perhaps
BDR glm(deaths ~ poly(age,2) + poly(age,1)*Smoke + offset(l),
BDR poisson)
BDR was your intention?
 In this parameterisation a 'poly(age,1)' term will appear among the
 coefficients with an estimated value of NA since it is aliased with
 'poly(age, 2)1'.  So I don't believe that this was Murray's intention.

 The only suggestion I can come up with is:

 summary(glm(cbind(deaths, l-deaths) ~ age*Smoke+I(age^2), family=binomial))

 [...]

 Coefficients:
  Estimate Std. Error z value Pr(|z|)
 (Intercept)  -10.798950.45149 -23.918   2e-16 ***
 age2.378920.20877  11.395   2e-16 ***
 SmokeYes   1.445730.37347   3.871 0.000108 ***
 I(age^2)  -0.197060.02749  -7.168  7.6e-13 ***
 age:SmokeYes  -0.308500.09756  -3.162 0.001566 **

 [...]

 Which doesn't use orthogonal polynomials anymore.  But I don't see how
 you can fit the model that Murray want to fit using orthogonal
 polynomials given the way R's model language operates.

 So I guess the Poisson GLM that Murray wants to fit is:

glm(deaths~ age*Smoke+I(age^2)+offset(l), family=poisson)

 Cheers,

Berwin

 == Full address 
 Berwin A Turlach  Tel.: +61 (8) 6488 3338 (secr)
 School of Mathematics and Statistics+61 (8) 6488 3383 (self)
 The University of Western Australia   FAX : +61 (8) 6488 1028
 35 Stirling Highway
 Crawley WA 6009e-mail: [EMAIL PROTECTED]
 Australiahttp://www.maths.uwa.edu.au/~berwin



-- 
Brian D. Ripley,  [EMAIL PROTECTED]
Professor of Applied Statistics,  http://www.stats.ox.ac.uk/~ripley/
University of Oxford, Tel:  +44 1865 272861 (self)
1 South Parks Road, +44 1865 272866 (PA)
Oxford OX1 3TG, UKFax:  +44 1865 272595

__
R-help@stat.math.ethz.ch mailing list
https://stat.ethz.ch/mailman/listinfo/r-help
PLEASE do read the posting guide http://www.R-project.org/posting-guide.html
and provide commented, minimal, self-contained, reproducible code.


Re: [R] Correspondence analysis with R -follow up

2006-07-21 Thread bady
Hi, Hi all,

 thank you for your answers; i've tried both cca from vegan library, and
 dudi.coa from ade4 library; one last question: my deal is mainly with
 contingency tables, like the one i'm posting here

 acciaieria-c(.41,.02,.44,.04,.09)
 laminatoio-c(.34,.28,.26,.01,.11)
 fonderia-c(.48,.05,.34,.08,.05)
 leghe-c(.45,.19,.25,.03,.08)
 pct-cbind(acciaieria,laminatoio,fonderia,leghe)

pct-data.frame(pct,row.names=c(normale,preparaz.,manutenz.,installaz.,trasferim.))


the data.frame pct is not a contingency table.

 BUT...cca and dudi.coa seem to return quite different graphical results;
 where am i doing wrong?
 Do they do the same to you with the table above?

graphicals reults are similar.
see the argument method in the function scatter.coa (library ade4).
...
  method: an integer between 1 and 3
   1 Rows and columns with the coordinates of lambda variance
   2 Rows variance 1 and columns by averaging
   3 Columns variance 1 and rows by averaging
...

#example :
require(ade4)
data(rpjdl)
coa1 - dudi.coa(rpjdl$fau, scannf = FALSE, nf = 4)
require(vegan)
coa2 - cca(rpjdl$fau)

# biplot representations
par(mfrow=c(2,2))
plot(coa2,type=text)
? scatter.coa
scatter(coa1,method=1)
scatter(coa1,method=2)
scatter(coa1,method=3)


hope this help :)


Pierre

__
R-help@stat.math.ethz.ch mailing list
https://stat.ethz.ch/mailman/listinfo/r-help
PLEASE do read the posting guide http://www.R-project.org/posting-guide.html
and provide commented, minimal, self-contained, reproducible code.


Re: [R] failed installing rgl

2006-07-21 Thread Prof Brian D Ripley

On FC5:

roc% rpm -q --file /usr/include/GL/glu.h
mesa-libGLU-devel-6.4.2-6.FC5.3

Do check what the R-admin manual says about -devel RPMs.


On Thu, 20 Jul 2006, James Foadi wrote:


Dear all,
I have tried installing rgl with the usual command:

R CMD INSTALL rgl_0.67-2.tar.gz

Differently from what happened last time I have succesfully installed this
package, this time there was a failure:

...
...g++ -I/usr/lib/R/include -I/usr/lib/R/include -I -DHAVE_PNG_H
-I/usr/include/libpng12 -I/usr/local/include   -fpic  -O2 -g -pipe -Wall
-Wp,-D_FORTIFY_SOURCE=2 -fexceptions -fstack-protector
--param=ssp-buffer-size=4 -m32 -march=i386 -mtune=generic
-fasynchronous-unwind-tables -c api.cpp -o api.o
In file included from glgui.hpp:9,
from gui.hpp:11,
from rglview.h:10,
from Device.hpp:11,
from DeviceManager.hpp:9,
from api.cpp:14:
opengl.hpp:23:20: error: GL/glu.h: No such file or directory
Disposable.hpp:13: warning: ÿÿstruct IDisposeListenerÿÿ has virtual functions
but non-virtual destructor
types.h:77: warning: ÿÿclass DestroyHandlerÿÿ has virtual functions but
non-virtual destructor
gui.hpp:56: warning: ÿÿclass gui::WindowImplÿÿ has virtual functions but
non-virtual destructor
gui.hpp:90: warning: ÿÿclass gui::GUIFactoryÿÿ has virtual functions but
non-virtual destructor
pixmap.h:39: warning: ÿÿclass PixmapFormatÿÿ has virtual functions but
non-virtual destructor
api.cpp: In function ÿÿvoid rgl_user2window(int*, int*, double*, double*,
double*, double*, int*)ÿÿ:
api.cpp:707: error: ÿÿgluProjectÿÿ was not declared in this scope
api.cpp: In function ÿÿvoid rgl_window2user(int*, int*, double*, double*,
double*, double*, int*)ÿÿ:
api.cpp:735: error: ÿÿgluUnProjectÿÿ was not declared in this scope
make: *** [api.o] Error 1
chmod: cannot access `/usr/lib/R/library/rgl/libs/*': No such file or
directory
ERROR: compilation failed for package 'rgl'
** Removing '/usr/lib/R/library/rgl'
...
...

It is clear that glu.h cannot be found during installation and, indeed, I have
checked and there is no glu.h under /usr/include/GL.

Any suggestion on how I could proceed?
I am using FEDORA CORE 5.
By typing glxinfo it seems that glu 1.3 is installed. But where's glu.h, then?

Many thanks

J

--
Dr James Foadi
Department of Physics
University of York
York YO10 5DD

email: [EMAIL PROTECTED]
Tel: 0044 1904 434622
Mobile: 0044 7740 678548


___
All New Yahoo! Mail ? Tired of [EMAIL PROTECTED]@! come-ons? Let our SpamGuard 
protect you. http://uk.docs.yahoo.com/nowyoucan.html




--
Brian D. Ripley,  [EMAIL PROTECTED]
Professor of Applied Statistics,  http://www.stats.ox.ac.uk/~ripley/
University of Oxford, Tel:  +44 1865 272861 (self)
1 South Parks Road, +44 1865 272866 (PA)
Oxford OX1 3TG, UKFax:  +44 1865 272595__
R-help@stat.math.ethz.ch mailing list
https://stat.ethz.ch/mailman/listinfo/r-help
PLEASE do read the posting guide http://www.R-project.org/posting-guide.html
and provide commented, minimal, self-contained, reproducible code.


Re: [R] Loss of numerical precision from conversion to list ?

2006-07-21 Thread Fabian Scheipl
Thank you both very much for your help.

Peter Dalgaard is right- i  didn't consider the fact that elementwise 
multiplication is column-wise rather than row-wise.
Sorry for taking up timespace with such a trivial mistake.



 Original-Nachricht 
Datum: 21 Jul 2006 10:07:31 +0200
Von: Peter Dalgaard [EMAIL PROTECTED]
An: Duncan Murdoch [EMAIL PROTECTED]
Betreff: Re: [R] Loss of numerical precision from conversion to list ?

 Duncan Murdoch [EMAIL PROTECTED] writes:
 
  R tries to use the maximum precision (64 bit mantissa) in the floating 
 ...
  Or perhaps your problem has nothing to do with this; I didn't really 
  look at it in detail.
 
 It hasn't. I was off speculating about sum vs rowSums too, but:
 
   num.v-  rowSums(((lambda-lambda0)*mu*w.k.sq[,-(K+1)])/(1+lambda*mu))
 
 Inside this, we have mu*w.k.sq[,-(K+1)] . mu is a vector of length 27,
 and w.k.sq has 10 rows and 28 *columns*. Column-major storage and
 vector recycling kicks in... If mu has identical elements (never mind
 the magnitude), of course, the recycling doesn't matter.
 
 -- 
O__   Peter Dalgaard Øster Farimagsgade 5, Entr.B
   c/ /'_ --- Dept. of Biostatistics PO Box 2099, 1014 Cph. K
  (*) \(*) -- University of Copenhagen   Denmark  Ph:  (+45)
 35327918
 ~~ - ([EMAIL PROTECTED])  FAX: (+45)
 35327907

-- 


Echte DSL-Flatrate dauerhaft für 0,- Euro*!

__
R-help@stat.math.ethz.ch mailing list
https://stat.ethz.ch/mailman/listinfo/r-help
PLEASE do read the posting guide http://www.R-project.org/posting-guide.html
and provide commented, minimal, self-contained, reproducible code.


Re: [R] failed installing rgl

2006-07-21 Thread James Foadi

Subject: Re: [R] failed installing rgl
Date: Friday 21 July 2006 10:08
From: Prof Brian D Ripley [EMAIL PROTECTED]
To: James Foadi [EMAIL PROTECTED]
Cc: r-help@stat.math.ethz.ch

On FC5:

roc% rpm -q --file /usr/include/GL/glu.h
mesa-libGLU-devel-6.4.2-6.FC5.3

Do check what the R-admin manual says about -devel RPMs.


I have followed Brian Ripley advice to read the R-admin manual. In Appendix A 
it is reported:

Remember that some package management systems (such as RPM and deb) make a 
distinction between the user version of a package and the development 
version. The latter usually has the same name but with the extension `-devel' 
or `-dev': you need both versions installed.

In fact file glu.h was not installed because only the user version of GLU 
was installed on my system. Thus, I have downloaded the development version 
of GLU (mesa-libGLU-devel-6.4.2-6.FC5.3.i386.rpm) and installed it.  After 
that the package rgl has installed fine.

Thank you, Brian.

J

---

-- 
Dr James Foadi
Department of Physics
University of York
York YO10 5DD

email: [EMAIL PROTECTED]
Tel: 0044 1904 434622
Mobile: 0044 7740 678548

__
R-help@stat.math.ethz.ch mailing list
https://stat.ethz.ch/mailman/listinfo/r-help
PLEASE do read the posting guide http://www.R-project.org/posting-guide.html
and provide commented, minimal, self-contained, reproducible code.


[R] from character to numeric over multiple columns

2006-07-21 Thread Federico Calboli
Hi All,

I have a data frame of three columns, all of which names. The names in the 
three 
  cols overlap up to a point, so I might have *Harry* in all three columns, 
*Tom* in cols 1 and 3 and *Bob* in col 3 only.

harry   paulbob
anita   harry   tom
frank   jackharry
tom peteben


I want to turn the names into numbers, BUT I want the numeric code for, say, 
*Harry*, to be the same on all columns.

Doing

cbind(as.numeric(col1), as.numeric(col2), as.numeric(col3))

does not work because the factors are different in each column, hence they get 
a 
different number even though the name is the same.

Ideas?

Cheers

Federico

-- 
Federico C. F. Calboli
Department of Epidemiology and Public Health
Imperial College, St Mary's Campus
Norfolk Place, London W2 1PG

Tel  +44 (0)20 7594 1602 Fax (+44) 020 7594 3193

f.calboli [.a.t] imperial.ac.uk
f.calboli [.a.t] gmail.com

__
R-help@stat.math.ethz.ch mailing list
https://stat.ethz.ch/mailman/listinfo/r-help
PLEASE do read the posting guide http://www.R-project.org/posting-guide.html
and provide commented, minimal, self-contained, reproducible code.


[R] [Fwd: Re: Parameterization puzzle]

2006-07-21 Thread Murray Jorgensen
Bother! This cold has made me accident-prone. I meant to hit Reply-all.
Clarification below.

 Original Message 
Subject: Re: [R] Parameterization puzzle
Date: Fri, 21 Jul 2006 19:10:03 +1200
From: Murray Jorgensen [EMAIL PROTECTED]
To: Prof Brian Ripley [EMAIL PROTECTED]
References: [EMAIL PROTECTED] 
[EMAIL PROTECTED]

Apologies for a non-selfcontained example. Here is what I should have sent:

pyears - scan()
18793 52407 10673 43248 5710 28612 2585 12663 1462 5317

deaths - scan()
2 32 12 104 28 206 28 186 31 102

l - log(pyears)
Smoke - gl(2,1,10,labels=c(No,Yes))
Age - gl(5,2,10,labels=c(35-44,45-54,55-64,65-74,75-84),
ordered=TRUE)
mod1.glm - glm(deaths ~ Age * Smoke + offset(l),family=poisson)
summary(mod1.glm)
age - as.numeric(Age)
mod2.glm - aso1.glm - glm(deaths ~ poly(age,2) + Smoke +
   poly(age,1):Smoke + offset(l),family=poisson)
summary(mod2.glm)


Cheers, Murray Jorgensen

Prof Brian Ripley wrote:
 R does not know that poly(age,2) and poly(age,1) are linearly dependent.
 (And indeed they only are for some functions 'poly'.)
 
 I cannot reproduce your example ('l' is missing), but perhaps
 
 glm(deaths ~ poly(age,2) + poly(age,1)*Smoke + offset(l), poisson)
 
 was your intention?
 
 On Fri, 21 Jul 2006, Murray Jorgensen wrote:
 
 Consider the following example (based on an example in Pat Altham's GLM 
 notes)

 pyears - scan()
 18793 52407 10673 43248 5710 28612 2585 12663 1462 5317

 deaths - scan()
 2 32 12 104 28 206 28 186 31 102

 Smoke - gl(2,1,10,labels=c(No,Yes))
 Age - gl(5,2,10,labels=c(35-44,45-54,55-64,65-74,75-84),
 ordered=TRUE)
 mod1.glm - glm(deaths ~ Age * Smoke + offset(l),family=poisson)
 summary(mod1.glm)
 age - as.numeric(Age)
 mod2.glm - aso1.glm - glm(deaths ~ poly(age,2) + Smoke +
poly(age,1):Smoke + offset(l),family=poisson)
 summary(mod2.glm)



 The business part of the summary for the first model

 Estimate Std. Error z value Pr(|z|)
 (Intercept)-5.927060.16577 -35.754   2e-16 ***
 Age.L   4.064900.47414   8.573   2e-16 ***
 Age.Q  -1.082930.41326  -2.620 0.008781 **
 Age.C   0.241580.31756   0.761 0.446816
 Age^4   0.042440.23061   0.184 0.853986
 SmokeYes0.619160.17296   3.580 0.000344 ***
 Age.L:SmokeYes -1.312340.49267  -2.664 0.007729 **
 Age.Q:SmokeYes  0.390430.43008   0.908 0.363976
 Age.C:SmokeYes -0.295930.33309  -0.888 0.374298
 Age^4:SmokeYes -0.036820.24432  -0.151 0.880218

 inspires me to fit the second model that omits the nonsignificant terms, 
 however this produces the summary

Estimate Std. Error z value Pr(|z|)
 (Intercept)-5.8368 0.1213 -48.103   2e-16 ***
 poly(age, 2)1   3.9483 0.1755  22.497   2e-16 ***
 poly(age, 2)2  -1.0460 0.1448  -7.223 5.08e-13 ***
 SmokeYes0.5183 0.1262   4.106 4.02e-05 ***
 SmokeNo:poly(age, 1)1.3755 0.4340   3.169  0.00153 **
 SmokeYes:poly(age, 1)   NA NA  NA   NA

 Why do we get a SmokeNo:poly(age, 1) term? Can I re-express mod2.glm so 
 that this term does not appear?

 Cheers,  Murray Jorgensen


 

-- 
Dr Murray Jorgensen  http://www.stats.waikato.ac.nz/Staff/maj.html
Department of Statistics, University of Waikato, Hamilton, New Zealand
Email: [EMAIL PROTECTED]Fax 7 838 4155
Phone  +64 7 838 4773 wkHome +64 7 825 0441Mobile 021 1395 862

-- 
Dr Murray Jorgensen  http://www.stats.waikato.ac.nz/Staff/maj.html
Department of Statistics, University of Waikato, Hamilton, New Zealand
Email: [EMAIL PROTECTED]Fax 7 838 4155
Phone  +64 7 838 4773 wkHome +64 7 825 0441Mobile 021 1395 862

__
R-help@stat.math.ethz.ch mailing list
https://stat.ethz.ch/mailman/listinfo/r-help
PLEASE do read the posting guide http://www.R-project.org/posting-guide.html
and provide commented, minimal, self-contained, reproducible code.


[R] [R-pkgs] tsDyn and RTisean packages on CRAN

2006-07-21 Thread Antonio, Fabio Di Narzo
Dear R users,
I've just uploaded  2 packages on CRAN, RTisean and tsDyn, both for time
series analysis (joint research projects with members of the Statistics
Department, University of Udine). Brief descriptions follow.

RTisean is an R interface to TISEAN executables
(http://www.mpipks-dresden.mpg.de/~tisean/). TISEAN is a suite of C
and Fortran routines for nonlinear time series analysis, coded and
documented by Rainer Hegger, Holger Kantz and Thomas Schreiber. In
RTisean, almost all TISEAN routines are wrapped in a conventional R
function (which silently calls TISEAN executables), with online help
and examples (thanks to Gianluca Gazzola).

tsDyn is a package for nonlinear time series modelling. At this point
the package focuses on Nonlinear Autoregressive Models, often indicated as
NLAR (as a major reference, see Tong (1990)). Currently available are
threshold (SETAR and LSTAR), neural networks (NNET), and additive
autoregressive (AAR) models. An experimental cross-platform tcltk GUI is
included for model selection. Explorative and diagnostic tools are
also available. A vignette is included for a clearer presentation of the
package contents.

Comments and suggestions appreciated

Antonio, Fabio Di Narzo
Dipartimento di Statistica
Università degli studi di Bologna.

___
R-packages mailing list
[EMAIL PROTECTED]
https://stat.ethz.ch/mailman/listinfo/r-packages

__
R-help@stat.math.ethz.ch mailing list
https://stat.ethz.ch/mailman/listinfo/r-help
PLEASE do read the posting guide http://www.R-project.org/posting-guide.html
and provide commented, minimal, self-contained, reproducible code.


Re: [R] from character to numeric over multiple columns

2006-07-21 Thread Prof Brian Ripley
Are the columns factors or character?  I'll try to write code that copes 
with both:

nm - unique(c(as.character(col1), as.character(col2), as.character(col3)))

DF[] - lapply(DF, function(x) match(x, nm))


On Fri, 21 Jul 2006, Federico Calboli wrote:

 Hi All,
 
 I have a data frame of three columns, all of which names. The names in the 
 three 
   cols overlap up to a point, so I might have *Harry* in all three columns, 
 *Tom* in cols 1 and 3 and *Bob* in col 3 only.
 
 harry paulbob
 anita harry   tom
 frank jackharry
 tom   peteben
   
 
 I want to turn the names into numbers, BUT I want the numeric code for, say, 
 *Harry*, to be the same on all columns.
 
 Doing
 
 cbind(as.numeric(col1), as.numeric(col2), as.numeric(col3))
 
 does not work because the factors are different in each column, hence they 
 get a 
 different number even though the name is the same.
 
 Ideas?
 
 Cheers
 
 Federico
 
 

-- 
Brian D. Ripley,  [EMAIL PROTECTED]
Professor of Applied Statistics,  http://www.stats.ox.ac.uk/~ripley/
University of Oxford, Tel:  +44 1865 272861 (self)
1 South Parks Road, +44 1865 272866 (PA)
Oxford OX1 3TG, UKFax:  +44 1865 272595

__
R-help@stat.math.ethz.ch mailing list
https://stat.ethz.ch/mailman/listinfo/r-help
PLEASE do read the posting guide http://www.R-project.org/posting-guide.html
and provide commented, minimal, self-contained, reproducible code.


Re: [R] from character to numeric over multiple columns

2006-07-21 Thread Federico Calboli
Prof Brian Ripley wrote:
 Are the columns factors or character?  I'll try to write code that copes 
 with both:
 
 nm - unique(c(as.character(col1), as.character(col2), as.character(col3)))
 
 DF[] - lapply(DF, function(x) match(x, nm))

Cheers,

it worked.

Federico

-- 
Federico C. F. Calboli
Department of Epidemiology and Public Health
Imperial College, St Mary's Campus
Norfolk Place, London W2 1PG

Tel  +44 (0)20 7594 1602 Fax (+44) 020 7594 3193

f.calboli [.a.t] imperial.ac.uk
f.calboli [.a.t] gmail.com

__
R-help@stat.math.ethz.ch mailing list
https://stat.ethz.ch/mailman/listinfo/r-help
PLEASE do read the posting guide http://www.R-project.org/posting-guide.html
and provide commented, minimal, self-contained, reproducible code.


[R] glm cannot find valid starting values

2006-07-21 Thread Dan Bebber
glm(S ~ -1 + Mdif, family=quasipoisson(link=identity), start=strt, sdat)
gives error:

 Error in glm.fit(x = X, y = Y, weights = weights, start = start, etastart 
 =
 etastart,  :
cannot find valid starting values: please specify some

strt is set to be the coefficient for a similar fit
glm(S ~ -1 + I(Mdif + 1),...
i.e. (Mdif + 1) is a vector similar to Mdif.
The error appears to occur when some values of Mdif are negative,
though I have not had this problem with simulated datasets.

Any solutions greatly appreciated (full details and data below).

Dan Bebber
Department of Plant Sciences
University of Oxford

OS: WinXP Pro SP2 and Win ME (tried both)
Processor: Dual Intel Xeon and Pentium 4 (tried both)
R version: 2.3.0 and 2.3.1 (tried both)

#Full details (can be pasted directly into R):
#Data:
sdat - data.frame(
S = c(0, 0, 0, 0, 28, 0, 1, 7, 0, 0, 39, 2, 0, 0, 40, 0, 0, 0, 0,
0, 0, 15, 0, 0, 3, 0, 0, 0, 2, 0, 3, 0, 30, 0, 20, 0, 1, 0, 0,
1, 21, 0, 0, 4, 14, 0, 0, 0, 1, 1, 1, 0, 0, 0, 1, 1, 0, 3, 0,
2, 5, 0, 0, 0, 0, 0, 0, 0, 25, 0, 5, 0, 0, 0, 1, 0, 1, 0, 0,
0, 0, 0, 0, 1, 0, 0, 0, 0, 1, 0, 1, 0, 0, 0, 0, 0, 1, 0, 0, 0,
0, 0, 0, 0, 0, 1, 10, 0, 0, 0, 0, 0, 9, 1, 1, 1, 1, 0, 0, 3,
0, 27, 7, 0, 0, 0, 0, 0, 1, 1, 4, 2, 1, 2, 4, 1, 4, 6, 12, 4,
6, 3, 4, 0, 4, 0, 6, 1, 3, 1, 4, 4, 1, 1, 2, 1, 6, 1, 0, 0, 1,
0, 1, 0, 0, 6, 0, 0, 0, 0, 2, 2, 3, 1, 6, 2, 2, 1, 1, 4, 4, 3,
3, 7, 1, 3, 5, 6, 4, 0, 1, 4, 2, 0, 1, 1, 1, 0, 1, 1, 1, 0, 0,
1, 0, 4, 0, 0, 1, 0, 2, 0, 0, 1, 2, 0, 4, 0, 2, 0, 3, 0, 2, 2,
0, 4, 0, 2, 0, 1, 2, 3, 0, 0, 0, 0, 3, 1, 0, 0, 0, 3, 2, 1, 2,
1, 1, 2, 0, 0, 3, 3, 1, 0, 1, 1, 2, 0, 1, 0, 1, 0, 1, 3, 2, 0,
0, 2, 1, 0, 1, 0, 0, 0, 0, 0, 0, 4, 2, 4, 0, 2, 0, 0, 0, 0, 0,
0, 1, 0, 0, 0, 0, 0, 1, 1, 0, 0, 0, 0, 1, 3, 2, 0, 0, 0, 0, 0,
0, 1, 0, 1, 0, 0, 0, 0, 0, 0, 0, 0, 1, 0, 0, 4, 1, 0, 0, 1),
M = 620+c(0,cumsum(sdat$S[-328])))
#S is the (unknown) number of N individuals that irreversibly change state 
in a time
#interval t. The data provided are a subset of the full data set.
#M is the cumulative sum of individuals that have changed state up to t-1.
#Assume that the rate of state change is constant (S ~ kM), but the
#distribution of S is clustered.
#The goal is to estimate N.
#N can be estimated by fitting:
qpglm - glm(S ~ M, family = quasipoisson(link = identity), sdat)
summary(qpglm)
N.est - -coef(qpglm)[1]/coef(qpglm)[2]
N.est
#i.e. x-intercept is minus intercept/slope
#To estimate confidence limits on N.est, fit models without intercept to
#N.est - M + x, where x is an integer. The model will have the lowest 
deviance
#when x = 0.
x - 0
Mdif - N.est - M + x
qpglm2 - glm(S ~ -1 + Mdif, family = quasipoisson(link = identity), sdat)
summary(qpglm2)
#Use analysis of deviance to estimate confidence limits on N.est:
disp - summary(qpglm)$dispersion
dfres - df.residual(qpglm)
dev.res - deviance(qpglm)
#From MASS4, p. 210, assume that changes in deviance scaled by
#dispersion as |x| increases have an F distribution
dev.crit - dev.res+qf(0.95,1,dfres)*disp
dev.crit
#values of x for which the deviance = dev.crit give approximate 95% 
confidence limits
#on N.est.
#The error occurs when x = -91.7:
x - -91.7
sdat$Mdif - N.est - sdat$M + x
strt - coef(glm(S ~ -1 + I(Mdif+1), family = quasipoisson(link = identity), 
sdat))
qpglm2 - glm(S ~ -1 + Mdif, family = quasipoisson(link = identity), 
start=strt, sdat)
#The problem is that this interferes with optimization to find values of x 
for which
#deviance = dev.crit

__
R-help@stat.math.ethz.ch mailing list
https://stat.ethz.ch/mailman/listinfo/r-help
PLEASE do read the posting guide http://www.R-project.org/posting-guide.html
and provide commented, minimal, self-contained, reproducible code.


[R] unexpected results

2006-07-21 Thread nathan

Hi, 

I'm just learning R and am encountering some unexpected results following a
guide on the internet. If anyone can help that would be great - I think it
is something about the way the data has been read in!

I've read a coma delimited text data file that was saved from excel:

 jacs.data - read.table(/Users/natha/Desktop/JACSdata2.txt, header=TRUE,
 sep=,)

This seemed to work fine, but then I start to get unusual results that don't
seem right:

The guide says that names(file.data) will give output like ID JURIS
RESCODE , but I get ID.JURIS.RESCODE.

The guide says that file.data[5,1] will give me the data for the 5th subject
but i get: 
[1] 7\tACT\t\t\tSUMMONS\tACTCC321.001\tA.C.T. - MINOR THEFT (REPLACEMENT
VALUE $2000 OR LESS)\ etc - which seems scrambled

The guide says that file.data[var50] will give me the data for all subject
who meet that condition (ie greater than 0 on var5), but I get:

Error in [.data.frame(jacs.data, offend  0) : 
object offend not found

can anyone help? Thanks nathan
-- 
View this message in context: 
http://www.nabble.com/unexpected-results-tf1979786.html#a5432210
Sent from the R help forum at Nabble.com.

__
R-help@stat.math.ethz.ch mailing list
https://stat.ethz.ch/mailman/listinfo/r-help
PLEASE do read the posting guide http://www.R-project.org/posting-guide.html
and provide commented, minimal, self-contained, reproducible code.


[R] rpart unbalanced data

2006-07-21 Thread helen . mills
Hello all,
I am currently working with rpart to classify vegetation types by spectral
characteristics, and am comming up with poor classifications based on the fact
that I have some vegetation types that have only 15 observations, while others
have over 100. I have attempted to supply prior weights to the dataset, though
this does not improve the classification greatly. Could anyone supply some
hints about how to improve a classification for a badly unbalanced datase?

Thank you,
Helen Mills Poulos

__
R-help@stat.math.ethz.ch mailing list
https://stat.ethz.ch/mailman/listinfo/r-help
PLEASE do read the posting guide http://www.R-project.org/posting-guide.html
and provide commented, minimal, self-contained, reproducible code.


Re: [R] glm cannot find valid starting values

2006-07-21 Thread Prof Brian Ripley
On Fri, 21 Jul 2006, Dan Bebber wrote:

 glm(S ~ -1 + Mdif, family=quasipoisson(link=identity), start=strt, sdat)
 gives error:
 
  Error in glm.fit(x = X, y = Y, weights = weights, start = start, etastart 
  =
  etastart,  :
 cannot find valid starting values: please specify some
 
 strt is set to be the coefficient for a similar fit
 glm(S ~ -1 + I(Mdif + 1),...
 i.e. (Mdif + 1) is a vector similar to Mdif.
 The error appears to occur when some values of Mdif are negative,
 though I have not had this problem with simulated datasets.

Right: if Mdif contains both positive and negative values there are no
coefficients that are valid for that model (you are bound to predict 
negative means).

You often do better to take etastart from another fit than start, but that 
will not help here, I believe.

BTW, your example cannot be pasted in as 'sdat' self-references.  It could 
be fixed, but I gave up at that point.


 
 Any solutions greatly appreciated (full details and data below).
 
 Dan Bebber
 Department of Plant Sciences
 University of Oxford
 
 OS: WinXP Pro SP2 and Win ME (tried both)
 Processor: Dual Intel Xeon and Pentium 4 (tried both)
 R version: 2.3.0 and 2.3.1 (tried both)
 
 #Full details (can be pasted directly into R):
 #Data:
 sdat - data.frame(
 S = c(0, 0, 0, 0, 28, 0, 1, 7, 0, 0, 39, 2, 0, 0, 40, 0, 0, 0, 0,
 0, 0, 15, 0, 0, 3, 0, 0, 0, 2, 0, 3, 0, 30, 0, 20, 0, 1, 0, 0,
 1, 21, 0, 0, 4, 14, 0, 0, 0, 1, 1, 1, 0, 0, 0, 1, 1, 0, 3, 0,
 2, 5, 0, 0, 0, 0, 0, 0, 0, 25, 0, 5, 0, 0, 0, 1, 0, 1, 0, 0,
 0, 0, 0, 0, 1, 0, 0, 0, 0, 1, 0, 1, 0, 0, 0, 0, 0, 1, 0, 0, 0,
 0, 0, 0, 0, 0, 1, 10, 0, 0, 0, 0, 0, 9, 1, 1, 1, 1, 0, 0, 3,
 0, 27, 7, 0, 0, 0, 0, 0, 1, 1, 4, 2, 1, 2, 4, 1, 4, 6, 12, 4,
 6, 3, 4, 0, 4, 0, 6, 1, 3, 1, 4, 4, 1, 1, 2, 1, 6, 1, 0, 0, 1,
 0, 1, 0, 0, 6, 0, 0, 0, 0, 2, 2, 3, 1, 6, 2, 2, 1, 1, 4, 4, 3,
 3, 7, 1, 3, 5, 6, 4, 0, 1, 4, 2, 0, 1, 1, 1, 0, 1, 1, 1, 0, 0,
 1, 0, 4, 0, 0, 1, 0, 2, 0, 0, 1, 2, 0, 4, 0, 2, 0, 3, 0, 2, 2,
 0, 4, 0, 2, 0, 1, 2, 3, 0, 0, 0, 0, 3, 1, 0, 0, 0, 3, 2, 1, 2,
 1, 1, 2, 0, 0, 3, 3, 1, 0, 1, 1, 2, 0, 1, 0, 1, 0, 1, 3, 2, 0,
 0, 2, 1, 0, 1, 0, 0, 0, 0, 0, 0, 4, 2, 4, 0, 2, 0, 0, 0, 0, 0,
 0, 1, 0, 0, 0, 0, 0, 1, 1, 0, 0, 0, 0, 1, 3, 2, 0, 0, 0, 0, 0,
 0, 1, 0, 1, 0, 0, 0, 0, 0, 0, 0, 0, 1, 0, 0, 4, 1, 0, 0, 1),
 M = 620+c(0,cumsum(sdat$S[-328])))
 #S is the (unknown) number of N individuals that irreversibly change state 
 in a time
 #interval t. The data provided are a subset of the full data set.
 #M is the cumulative sum of individuals that have changed state up to t-1.
 #Assume that the rate of state change is constant (S ~ kM), but the
 #distribution of S is clustered.
 #The goal is to estimate N.
 #N can be estimated by fitting:
 qpglm - glm(S ~ M, family = quasipoisson(link = identity), sdat)
 summary(qpglm)
 N.est - -coef(qpglm)[1]/coef(qpglm)[2]
 N.est
 #i.e. x-intercept is minus intercept/slope
 #To estimate confidence limits on N.est, fit models without intercept to
 #N.est - M + x, where x is an integer. The model will have the lowest 
 deviance
 #when x = 0.
 x - 0
 Mdif - N.est - M + x
 qpglm2 - glm(S ~ -1 + Mdif, family = quasipoisson(link = identity), sdat)
 summary(qpglm2)
 #Use analysis of deviance to estimate confidence limits on N.est:
 disp - summary(qpglm)$dispersion
 dfres - df.residual(qpglm)
 dev.res - deviance(qpglm)
 #From MASS4, p. 210, assume that changes in deviance scaled by
 #dispersion as |x| increases have an F distribution
 dev.crit - dev.res+qf(0.95,1,dfres)*disp
 dev.crit
 #values of x for which the deviance = dev.crit give approximate 95% 
 confidence limits
 #on N.est.
 #The error occurs when x = -91.7:
 x - -91.7
 sdat$Mdif - N.est - sdat$M + x
 strt - coef(glm(S ~ -1 + I(Mdif+1), family = quasipoisson(link = identity), 
 sdat))
 qpglm2 - glm(S ~ -1 + Mdif, family = quasipoisson(link = identity), 
 start=strt, sdat)
 #The problem is that this interferes with optimization to find values of x 
 for which
 #deviance = dev.crit
 
 __
 R-help@stat.math.ethz.ch mailing list
 https://stat.ethz.ch/mailman/listinfo/r-help
 PLEASE do read the posting guide http://www.R-project.org/posting-guide.html
 and provide commented, minimal, self-contained, reproducible code.
 

-- 
Brian D. Ripley,  [EMAIL PROTECTED]
Professor of Applied Statistics,  http://www.stats.ox.ac.uk/~ripley/
University of Oxford, Tel:  +44 1865 272861 (self)
1 South Parks Road, +44 1865 272866 (PA)
Oxford OX1 3TG, UKFax:  +44 1865 272595

__
R-help@stat.math.ethz.ch mailing list
https://stat.ethz.ch/mailman/listinfo/r-help
PLEASE do read the posting guide http://www.R-project.org/posting-guide.html
and provide commented, minimal, self-contained, reproducible code.


Re: [R] rpart unbalanced data

2006-07-21 Thread Dr. Diego Kuonen
Dear Helen,

You may want to have a look at

  http://www.togaware.com/datamining/survivor/Predicting_Fraud.html

Greets,

  Diego Kuonen


[EMAIL PROTECTED] wrote:
 Hello all,
 I am currently working with rpart to classify vegetation types by spectral
 characteristics, and am comming up with poor classifications based on the fact
 that I have some vegetation types that have only 15 observations, while others
 have over 100. I have attempted to supply prior weights to the dataset, though
 this does not improve the classification greatly. Could anyone supply some
 hints about how to improve a classification for a badly unbalanced datase?
 
 Thank you,
 Helen Mills Poulos

-- 
Dr. ès sc. Diego Kuonen, CEOphone  +41 (0)21 693 5508
Statoo Consulting   fax+41 (0)21 693 8765
PO Box 107  mobile +41 (0)78 709 5384
CH-1015 Lausanne 15 email   [EMAIL PROTECTED]
web   http://www.statoo.info   skype Kuonen.Statoo.Consulting
-
| Statistical Consulting + Data Analysis + Data Mining Services |
-
+  Are you drowning in information and starving for knowledge?  +
+  Have you ever been Statooed?  http://www.statoo.biz  +

__
R-help@stat.math.ethz.ch mailing list
https://stat.ethz.ch/mailman/listinfo/r-help
PLEASE do read the posting guide http://www.R-project.org/posting-guide.html
and provide commented, minimal, self-contained, reproducible code.


Re: [R] unexpected results

2006-07-21 Thread Duncan Murdoch
On 7/21/2006 7:36 AM, nathan wrote:
 Hi, 
 
 I'm just learning R and am encountering some unexpected results following a
 guide on the internet. If anyone can help that would be great - I think it
 is something about the way the data has been read in!
 
 I've read a coma delimited text data file that was saved from excel:
 
 jacs.data - read.table(/Users/natha/Desktop/JACSdata2.txt, header=TRUE,
 sep=,)
 
 This seemed to work fine, but then I start to get unusual results that don't
 seem right:
 
 The guide says that names(file.data) will give output like ID JURIS
 RESCODE , but I get ID.JURIS.RESCODE.
 
 The guide says that file.data[5,1] will give me the data for the 5th subject
 but i get: 
 [1] 7\tACT\t\t\tSUMMONS\tACTCC321.001\tA.C.T. - MINOR THEFT (REPLACEMENT
 VALUE $2000 OR LESS)\ etc - which seems scrambled

The \t values are tabs.  I think your file was tab delimited, not 
comma delimited.  R thinks it has only one column, because it didn't 
fine any commas.
 
 The guide says that file.data[var50] will give me the data for all subject
 who meet that condition (ie greater than 0 on var5), but I get:
 
 Error in [.data.frame(jacs.data, offend  0) : 
   object offend not found

It looks like you typed jacs.data[offend  0].  There are two problems:

1.  You want to select rows matching the condition, so you need another 
comma, i.e.

jacs.data[offend  0, ]

(the empty entry after the comma means all columns).

2.  You need to have a variable named offend outside the dataframe.  The 
error message makes it look as though you don't.

If offend is a column in the dataframe, then you would use

jacs.data[jacs.data$offend  0, ]

or

subset(jacs.data, offend  0)

Duncan Murdoch
 can anyone help? Thanks nathan

__
R-help@stat.math.ethz.ch mailing list
https://stat.ethz.ch/mailman/listinfo/r-help
PLEASE do read the posting guide http://www.R-project.org/posting-guide.html
and provide commented, minimal, self-contained, reproducible code.


Re: [R] unexpected results

2006-07-21 Thread Marc Schwartz
On Fri, 2006-07-21 at 04:36 -0700, nathan wrote:
 Hi, 
 
 I'm just learning R and am encountering some unexpected results following a
 guide on the internet. If anyone can help that would be great - I think it
 is something about the way the data has been read in!
 
 I've read a coma delimited text data file that was saved from excel:
 
  jacs.data - read.table(/Users/natha/Desktop/JACSdata2.txt, header=TRUE,
  sep=,)
 
 This seemed to work fine, but then I start to get unusual results that don't
 seem right:
 
 The guide says that names(file.data) will give output like ID JURIS
 RESCODE , but I get ID.JURIS.RESCODE.
 
 The guide says that file.data[5,1] will give me the data for the 5th subject
 but i get: 
 [1] 7\tACT\t\t\tSUMMONS\tACTCC321.001\tA.C.T. - MINOR THEFT (REPLACEMENT
 VALUE $2000 OR LESS)\ etc - which seems scrambled
 
 The guide says that file.data[var50] will give me the data for all subject
 who meet that condition (ie greater than 0 on var5), but I get:
 
 Error in [.data.frame(jacs.data, offend  0) : 
   object offend not found
 
 can anyone help? Thanks nathan

It would appear that your data file is NOT comma delimited, but TAB
delimited.

The \t characters in the output above support this, since you asked
for just the first column for the 5th subject and it appears that you
got the entire row.

Re-run the import using:

jacs.data - read.table(/Users/natha/Desktop/JACSdata2.txt,
header=TRUE, sep = \t)

so that you are indicating that the delimiter is a TAB character, not a
comma.

HTH,

Marc Schwartz

__
R-help@stat.math.ethz.ch mailing list
https://stat.ethz.ch/mailman/listinfo/r-help
PLEASE do read the posting guide http://www.R-project.org/posting-guide.html
and provide commented, minimal, self-contained, reproducible code.


Re: [R] unexpected results

2006-07-21 Thread Petr Pikal
Hi

On 21 Jul 2006 at 4:36, nathan wrote:

Date sent:  Fri, 21 Jul 2006 04:36:53 -0700 (PDT)
From:   nathan [EMAIL PROTECTED]
To: r-help@stat.math.ethz.ch
Subject:[R] unexpected results

 
 Hi, 
 
 I'm just learning R and am encountering some unexpected results
 following a guide on the internet. If anyone can help that would be
 great - I think it is something about the way the data has been read
 in!
 
 I've read a coma delimited text data file that was saved from excel:
 
  jacs.data - read.table(/Users/natha/Desktop/JACSdata2.txt,
  header=TRUE, sep=,)
 
 This seemed to work fine, but then I start to get unusual results that
I do not think so. Probably separation character of your file is not 
, as you set in sep=,.

 don't seem right:
 
 The guide says that names(file.data) will give output like ID
 JURIS RESCODE , but I get ID.JURIS.RESCODE.

e.g. ID.JURIS.RESCODE was read into one column.

Best way how to copy data from Excel (if you have Excel) 

Select your data in Excel including first row with headers
Ctrl-C
Go to R
Write mydata - read.delim(clipboard)


or look at JACSdata2.txt what is the separator between fields and set 
it in your read.table command accordingly. From later I suppose your 
txt file is tab delimited so
read.delim()
shall capture it.

HTH
Petr

 
 The guide says that file.data[5,1] will give me the data for the 5th
 subject but i get: [1] 7\tACT\t\t\tSUMMONS\tACTCC321.001\tA.C.T. -
 MINOR THEFT (REPLACEMENT VALUE $2000 OR LESS)\ etc - which seems
 scrambled
 The guide says that file.data[var50] will give me the data for all
 subject who meet that condition (ie greater than 0 on var5), but I
 get:
 
 Error in [.data.frame(jacs.data, offend  0) : 
  object offend not found
 
 can anyone help? Thanks nathan
 -- 
 View this message in context:
 http://www.nabble.com/unexpected-results-tf1979786.html#a5432210 Sent
 from the R help forum at Nabble.com.
 
 __
 R-help@stat.math.ethz.ch mailing list
 https://stat.ethz.ch/mailman/listinfo/r-help
 PLEASE do read the posting guide
 http://www.R-project.org/posting-guide.html and provide commented,
 minimal, self-contained, reproducible code.

Petr Pikal
[EMAIL PROTECTED]

__
R-help@stat.math.ethz.ch mailing list
https://stat.ethz.ch/mailman/listinfo/r-help
PLEASE do read the posting guide http://www.R-project.org/posting-guide.html
and provide commented, minimal, self-contained, reproducible code.


[R] Reading data with blank elements

2006-07-21 Thread Ahamarshan jn
Hi,
 I have a dataset saved in *.csv format, that contains
13 columns (the first column being the title name and
the rest experiments) and about 2500 rows.
Not all columns in the row have data in it
i.e for eg

BS00,-0.084,0.0136,-0.1569,-0.6484,1.103,1.7859,0.40287,0.5368,0.08461,-0.1935,-0.147974,0.30685

BS01,0.491270283,0.875826172,,

BS02,0.090794476,0.225858954,,,0.32643,0.34317,0.133145295,,,0.115832599,0.47636458,

BS03,0.019828221,-0.095735935,-0.122767219,-0.0676,0.002533,-0.1510361,0.736247,2.053192,-0.423658,0.4591219,1.1245015,

BS04,-0.435189342,-0.041595955,-0.781281128,-1.923036,-3.2301671020.152322609,-1.495513519,,


I am using R to perform a correlation, but I am
getting an error while trying to read the data as



person.data-read.table(datafile.csv,header=TRUE,sep=',',row.names=1)

Error in scan (file = file, what = what, sep = sep,
quote = quote, dec = dec,  : 
line 1919 did not have 13 elements
Execution halted 
 
The error looks as though there is a problem with the
last element being not read when it is blank. I could
introduce terms like na to the blank elements but I
donot want to do that because this will hinder my
future analysis. 

Can some one suggest me a solution to overcome this
problem while reading the data? , or is there
something that I have missed to make the data
readable. 

Thank you in advance, 

PS: The data was imported from a experiment and saved
in excel sheet as a *.csv and then used.

__
R-help@stat.math.ethz.ch mailing list
https://stat.ethz.ch/mailman/listinfo/r-help
PLEASE do read the posting guide http://www.R-project.org/posting-guide.html
and provide commented, minimal, self-contained, reproducible code.


Re: [R] Reading data with blank elements

2006-07-21 Thread jim holtman
Use

x - count.fields('datafile.csv',sep=',')

on your file.  This will tell you the number of columns that R thinks is one
each line.  Then do:

table(x)

to see if all the lines have the same number of columns.  If not, then find
the lines which don't:

which(x != legal.length)

and then look at your input data for those lines.




On 7/21/06, Ahamarshan jn [EMAIL PROTECTED] wrote:

 Hi,
 I have a dataset saved in *.csv format, that contains
 13 columns (the first column being the title name and
 the rest experiments) and about 2500 rows.
 Not all columns in the row have data in it
 i.e for eg

 BS00,-0.084,0.0136,-0.1569,-0.6484,1.103,1.7859,0.40287,0.5368,0.08461,-
 0.1935,-0.147974,0.30685

 BS01,0.491270283,0.875826172,,

 BS02,0.090794476,0.225858954,,,0.32643,0.34317,0.133145295,,,0.115832599,
 0.47636458,

 BS03,0.019828221,-0.095735935,-0.122767219,-0.0676,0.002533,-0.1510361,
 0.736247,2.053192,-0.423658,0.4591219,1.1245015,

 BS04,-0.435189342,-0.041595955,-0.781281128,-1.923036,-3.230167102
 0.152322609,-1.495513519,,


 I am using R to perform a correlation, but I am
 getting an error while trying to read the data as


 
 person.data-read.table(datafile.csv,header=TRUE,sep=',',row.names=1)

 Error in scan (file = file, what = what, sep = sep,
 quote = quote, dec = dec,  :
line 1919 did not have 13 elements
 Execution halted 

 The error looks as though there is a problem with the
 last element being not read when it is blank. I could
 introduce terms like na to the blank elements but I
 donot want to do that because this will hinder my
 future analysis.

 Can some one suggest me a solution to overcome this
 problem while reading the data? , or is there
 something that I have missed to make the data
 readable.

 Thank you in advance,

 PS: The data was imported from a experiment and saved
 in excel sheet as a *.csv and then used.

 __
 R-help@stat.math.ethz.ch mailing list
 https://stat.ethz.ch/mailman/listinfo/r-help
 PLEASE do read the posting guide
 http://www.R-project.org/posting-guide.html
 and provide commented, minimal, self-contained, reproducible code.




-- 
Jim Holtman
Cincinnati, OH
+1 513 646 9390

What is the problem you are trying to solve?

[[alternative HTML version deleted]]

__
R-help@stat.math.ethz.ch mailing list
https://stat.ethz.ch/mailman/listinfo/r-help
PLEASE do read the posting guide http://www.R-project.org/posting-guide.html
and provide commented, minimal, self-contained, reproducible code.


Re: [R] Reading data with blank elements

2006-07-21 Thread Gabor Grothendieck
See if this works:

read.csv(datafile.csv, row.names = 1, fill = TRUE)


On 7/21/06, Ahamarshan jn [EMAIL PROTECTED] wrote:
 Hi,
  I have a dataset saved in *.csv format, that contains
 13 columns (the first column being the title name and
 the rest experiments) and about 2500 rows.
 Not all columns in the row have data in it
 i.e for eg

 BS00,-0.084,0.0136,-0.1569,-0.6484,1.103,1.7859,0.40287,0.5368,0.08461,-0.1935,-0.147974,0.30685

 BS01,0.491270283,0.875826172,,

 BS02,0.090794476,0.225858954,,,0.32643,0.34317,0.133145295,,,0.115832599,0.47636458,

 BS03,0.019828221,-0.095735935,-0.122767219,-0.0676,0.002533,-0.1510361,0.736247,2.053192,-0.423658,0.4591219,1.1245015,

 BS04,-0.435189342,-0.041595955,-0.781281128,-1.923036,-3.2301671020.152322609,-1.495513519,,


 I am using R to perform a correlation, but I am
 getting an error while trying to read the data as


 
 person.data-read.table(datafile.csv,header=TRUE,sep=',',row.names=1)

 Error in scan (file = file, what = what, sep = sep,
 quote = quote, dec = dec,  :
line 1919 did not have 13 elements
 Execution halted 

 The error looks as though there is a problem with the
 last element being not read when it is blank. I could
 introduce terms like na to the blank elements but I
 donot want to do that because this will hinder my
 future analysis.

 Can some one suggest me a solution to overcome this
 problem while reading the data? , or is there
 something that I have missed to make the data
 readable.

 Thank you in advance,

 PS: The data was imported from a experiment and saved
 in excel sheet as a *.csv and then used.

 __
 R-help@stat.math.ethz.ch mailing list
 https://stat.ethz.ch/mailman/listinfo/r-help
 PLEASE do read the posting guide http://www.R-project.org/posting-guide.html
 and provide commented, minimal, self-contained, reproducible code.


__
R-help@stat.math.ethz.ch mailing list
https://stat.ethz.ch/mailman/listinfo/r-help
PLEASE do read the posting guide http://www.R-project.org/posting-guide.html
and provide commented, minimal, self-contained, reproducible code.


[R] positive semi-definite matrix

2006-07-21 Thread roger bos
I have a covariance matrix that is not positive semi-definite matrix and I
need it to be via some sort of adjustment.  Is there any R routine or
package to help me do this?

Thanks, Roger

[[alternative HTML version deleted]]

__
R-help@stat.math.ethz.ch mailing list
https://stat.ethz.ch/mailman/listinfo/r-help
PLEASE do read the posting guide http://www.R-project.org/posting-guide.html
and provide commented, minimal, self-contained, reproducible code.


Re: [R] glm cannot find valid starting values

2006-07-21 Thread Dan Bebber
Brian Ripley wrote:

 BTW, your example cannot be pasted in as 'sdat' self-references.  It could
 be fixed, but I gave up at that point.

Oh dear, I'm very sorry. I forgot to run rm(list=ls(all=TRUE)) before 
testing.
The corrected code is:

#Data:
S - c(0, 0, 0, 0, 28, 0, 1, 7, 0, 0, 39, 2, 0, 0, 40, 0, 0, 0, 0,
0, 0, 15, 0, 0, 3, 0, 0, 0, 2, 0, 3, 0, 30, 0, 20, 0, 1, 0, 0,
1, 21, 0, 0, 4, 14, 0, 0, 0, 1, 1, 1, 0, 0, 0, 1, 1, 0, 3, 0,
2, 5, 0, 0, 0, 0, 0, 0, 0, 25, 0, 5, 0, 0, 0, 1, 0, 1, 0, 0,
0, 0, 0, 0, 1, 0, 0, 0, 0, 1, 0, 1, 0, 0, 0, 0, 0, 1, 0, 0, 0,
0, 0, 0, 0, 0, 1, 10, 0, 0, 0, 0, 0, 9, 1, 1, 1, 1, 0, 0, 3,
0, 27, 7, 0, 0, 0, 0, 0, 1, 1, 4, 2, 1, 2, 4, 1, 4, 6, 12, 4,
6, 3, 4, 0, 4, 0, 6, 1, 3, 1, 4, 4, 1, 1, 2, 1, 6, 1, 0, 0, 1,
0, 1, 0, 0, 6, 0, 0, 0, 0, 2, 2, 3, 1, 6, 2, 2, 1, 1, 4, 4, 3,
3, 7, 1, 3, 5, 6, 4, 0, 1, 4, 2, 0, 1, 1, 1, 0, 1, 1, 1, 0, 0,
1, 0, 4, 0, 0, 1, 0, 2, 0, 0, 1, 2, 0, 4, 0, 2, 0, 3, 0, 2, 2,
0, 4, 0, 2, 0, 1, 2, 3, 0, 0, 0, 0, 3, 1, 0, 0, 0, 3, 2, 1, 2,
1, 1, 2, 0, 0, 3, 3, 1, 0, 1, 1, 2, 0, 1, 0, 1, 0, 1, 3, 2, 0,
0, 2, 1, 0, 1, 0, 0, 0, 0, 0, 0, 4, 2, 4, 0, 2, 0, 0, 0, 0, 0,
0, 1, 0, 0, 0, 0, 0, 1, 1, 0, 0, 0, 0, 1, 3, 2, 0, 0, 0, 0, 0,
0, 1, 0, 1, 0, 0, 0, 0, 0, 0, 0, 0, 1, 0, 0, 4, 1, 0, 0, 1)
M - 620+c(0,cumsum(S[-328]))
sdat - data.frame(S,M)
#S is the number of N individuals that irreversibly change state in a time
#interval t. The data provided are a subset of the full data set.
#M is the cumulative sum of individuals that have changed state up to t-1.
#Assume that the rate of state change is constant (S ~ kM), but the
#distribution of S is clustered.
#N can be estimated by fitting:
qpglm - glm(S ~ M, family = quasipoisson(link = identity), sdat)
summary(qpglm)
N.est - -coef(qpglm)[1]/coef(qpglm)[2]
N.est
#i.e. x-intercept is minus intercept/slope
#To estimate confidence limits on N.est, fit models without intercept to
#N.est - M + x, where x is an integer. The model will have the lowest 
deviance
#when x = 0.
x - 0
Mdif - N.est - M + x
qpglm2 - glm(S ~ -1 + Mdif, family = quasipoisson(link = identity), sdat)
summary(qpglm2)
#Use analysis of deviance to estimate confidence limits on N.est:
disp - summary(qpglm)$dispersion
dfres - df.residual(qpglm)
dev.res - deviance(qpglm)
#From MASS4, p. 210, assume that changes in deviance scaled by
#dispersion as |x| increases have an F distribution
dev.crit - dev.res+qf(0.95,1,dfres)*disp
dev.crit
#values of x for which the deviance = dev.crit give approximate 95% 
confidence limits
#on N.est.
#The error occurs when x = -91.7:
x - -91.7
sdat$Mdif - N.est - sdat$M + x
strt - coef(glm(S ~ -1 + I(Mdif+1), family = quasipoisson(link = identity), 
sdat))
qpglm2 - glm(S ~ -1 + Mdif, family = quasipoisson(link = identity), 
start=strt, sdat)

__
R-help@stat.math.ethz.ch mailing list
https://stat.ethz.ch/mailman/listinfo/r-help
PLEASE do read the posting guide http://www.R-project.org/posting-guide.html
and provide commented, minimal, self-contained, reproducible code.


Re: [R] Reading data with blank elements

2006-07-21 Thread Philipp Pagel

Hi!

On Fri, Jul 21, 2006 at 05:43:03AM -0700, Ahamarshan jn wrote:
  I have a dataset saved in *.csv format, that contains
[...]

 BS00,-0.084,0.0136,-0.1569,-0.6484,1.103,1.7859,0.40287,0.5368,0.08461,-0.1935,-0.147974,0.30685
 BS01,0.491270283,0.875826172,,
 BS02,0.090794476,0.225858954,,,0.32643,0.34317,0.133145295,,,0.115832599,0.47636458,
 BS03,0.019828221,-0.095735935,-0.122767219,-0.0676,0.002533,-0.1510361,0.736247,2.053192,-0.423658,0.4591219,1.1245015,
 BS04,-0.435189342,-0.041595955,-0.781281128,-1.923036,-3.2301671020.152322609,-1.495513519,,


 person.data-read.table(datafile.csv,header=TRUE,sep=',',row.names=1)
 
 Error in scan (file = file, what = what, sep = sep,
 quote = quote, dec = dec,  : 
 line 1919 did not have 13 elements
 Execution halted 

R does handle empty elements fine. The error message you quote occurs if
a row does not contain the expected number of elements (empty or not.)

Did you have a look at row 1919? Does it really contain the same number
of separators (commas) as the other ones?

Some programs handle empty elements at the end of a row in a 'lazy' way
and simply ommit them. If this is the case you can use the option
'fill=TRUE' to tell read.table that you want it to silently pad short
rows with empty elements.

Another 'popular' reason for funny errors with read.table is the
unexpected occurence of quotation or comment characters in the data...

cu
Philipp

-- 
Dr. Philipp PagelTel.  +49-8161-71 2131
Dept. of Genome Oriented Bioinformatics  Fax.  +49-8161-71 2186
Technical University of Munich
Science Center Weihenstephan
85350 Freising, Germany

 and

Institute for Bioinformatics / MIPS  Tel.  +49-89-3187 3675
GSF - National Research Center   Fax.  +49-89-3187 3585
  for Environment and Health
Ingolstädter Landstrasse 1
85764 Neuherberg, Germany
http://mips.gsf.de/staff/pagel

__
R-help@stat.math.ethz.ch mailing list
https://stat.ethz.ch/mailman/listinfo/r-help
PLEASE do read the posting guide http://www.R-project.org/posting-guide.html
and provide commented, minimal, self-contained, reproducible code.


Re: [R] positive semi-definite matrix

2006-07-21 Thread Duncan Murdoch
On 7/21/2006 8:59 AM, roger bos wrote:
 I have a covariance matrix that is not positive semi-definite matrix and I
 need it to be via some sort of adjustment.  Is there any R routine or
 package to help me do this?

I think you need to be more specific about what have and what you want, 
but if the matrix is symmetric and nearly positive semi-definite (but 
not exactly because of rounding error), you might try something like

fixit - function(A) {
   eig - eigen(A, symmetric = TRUE)
   eig$values - pmax(0, eig$values)
   return(eig$vectors %*% diag(eig$values) %*% t(eig$vectors))
}

Rounding error means this is not guaranteed to be positive 
semi-definite, but it will be very close.

Duncan Murdoch

__
R-help@stat.math.ethz.ch mailing list
https://stat.ethz.ch/mailman/listinfo/r-help
PLEASE do read the posting guide http://www.R-project.org/posting-guide.html
and provide commented, minimal, self-contained, reproducible code.


[R] (no subject)

2006-07-21 Thread Munoz-Melendez, Gabriela
Hi,
 
I am having problems when trying to apply factor analysis in a covariance 
matrix,
 
 factanal(covmat=strip1cmcd, factors=5, control=list(lower=0.0264))

Error in optim(start, FAfn, FAgr, method = L-BFGS-B, lower = lower,  : 
non-finite value supplied by optim
In addition: Warning message:
NaNs produced in: sqrt(diag(cv)) 

I have searched for possible solutions in messages posted previously. I think 
that: 
 
Zeroing the workspace is not a requirement of the original L-BFGS-B code 
that I can see.  Given that it was originally in Fortran, and Fortran 
often does zero it seems a likely symptom, but it does mean that a 
variable is being used uninitialized somewhere in the code (converted to 
C).  It would be better to leave vect alone and to zero the workspace with 
a memset call in lbfgsb.  (Incidentally, I don't know why S_alloc does not 
use memset -- we do require standard C and use in seeral other places.)

-- 
Brian D. Ripley,   
 
could be relevant to my case. The question is: How can I zero the workspace 
with a memset call in lbfgsb?
I have read the Rhelp for windows and tried to zero the workspace from 
RGui.exe mim.mem.size but is not working. Any suggestions will be very 
welcome indeed.
 
Regards
 
 
Dr. Gabriela Munoz-Melendez
Environmental Processes and Systems Research Group
 
Department of Earth Science and Engineering
Imperial College London
SW7 2AZ
 
Tel: +44 (0)207 594 7369 
Fax: +44 (0)207 594 7354
E-mail: [EMAIL PROTECTED]   

[[alternative HTML version deleted]]

__
R-help@stat.math.ethz.ch mailing list
https://stat.ethz.ch/mailman/listinfo/r-help
PLEASE do read the posting guide http://www.R-project.org/posting-guide.html
and provide commented, minimal, self-contained, reproducible code.


Re: [R] positive semi-definite matrix

2006-07-21 Thread Martin Maechler

 Duncan == Duncan Murdoch [EMAIL PROTECTED]
 on Fri, 21 Jul 2006 09:44:42 -0400 writes:

Duncan On 7/21/2006 8:59 AM, roger bos wrote:
 I have a covariance matrix that is not positive semi-definite matrix and 
I
 need it to be via some sort of adjustment.  Is there any R routine or
 package to help me do this?

Duncan I think you need to be more specific about what have and what you 
want, 
Duncan but if the matrix is symmetric and nearly positive semi-definite 
(but 
Duncan not exactly because of rounding error), you might try something like

Duncan fixit - function(A) {
Duncan eig - eigen(A, symmetric = TRUE)
Duncan eig$values - pmax(0, eig$values)
Duncan return(eig$vectors %*% diag(eig$values) %*% t(eig$vectors))
Duncan }

Duncan Rounding error means this is not guaranteed to be positive 
Duncan semi-definite, but it will be very close.

A slightly more general and stable version of the above
is available via  sfsmisc::posdefify(.) :

install.packages(sfsmisc)
?posdefify


Martin Maechler, ETH Zurich

__
R-help@stat.math.ethz.ch mailing list
https://stat.ethz.ch/mailman/listinfo/r-help
PLEASE do read the posting guide http://www.R-project.org/posting-guide.html
and provide commented, minimal, self-contained, reproducible code.


[R] : Arial font for text in lattice plots under Linux

2006-07-21 Thread Peter Ho
Hi,

I have been asked by a publisher to change the font style of a lattice 
plot in my manuscript. I have consulted documentation on trellis 
graphics and the excellent book R graphics, but am still not sure how 
I could create plots with Arial as the font style for text in the plot.  
I am running R (Version 2.3.1 (2006-06-01)) under debian Linux. I have 
msttcorefonts installed.

Will it be possible to do this with jpeg() as a device? Or with 
postscript()?




Peter

__
R-help@stat.math.ethz.ch mailing list
https://stat.ethz.ch/mailman/listinfo/r-help
PLEASE do read the posting guide http://www.R-project.org/posting-guide.html
and provide commented, minimal, self-contained, reproducible code.


Re: [R] Weibull distribution

2006-07-21 Thread Valentin Dimitrov
Dear Leaf,

I modified your code as follows:

gamma.fun - function(mu,sd,start=100)
 {
f.fn - function(alpha) 
{abs(sd^2-mu^2/(gamma(1+1/alpha))^2*(gamma(1+2/alpha)-(gamma(1+1/alpha))^2))}
alpha - optim(start, f.fn)
beta - mu/gamma(1+1/alpha$par)
return(list=c(a=alpha$par,b=beta));
 }

Now it works properly.

First, I added an abs(). You tried to solve an
equation by means of the R-function optim(), which
finds a minimum. That's why you can find the solution
of f(x)=a through minimization of abs(f(x)-a).
Second, I deleted the optim-method BFGS from the
optim() function, because it is not appropriate in
this case. BFGS seeks to make the gradient (or here
the first derivative) zero and in your case f(x)
converges to a constant for big x, which means f'(x)
is approximately 0 for big x, which is why BFGS stops
almost immediately after the start value. The default
method of optim() ( Nelder and Mead ) is more
appropriate, since it does not need the first
derivative and works only with function values.

Best regards,
Valentin


--- Leaf Sun [EMAIL PROTECTED] wrote:

 Hi all,
 
 By its definition, the mean and variance of two-par.
 Weibull distribution are:
 
  
 
  
 
  (www.wikipedia.org)
 
 
 I was wondering, if given mean and sd. could we
 parameterize the distribution? I tried this in R.
 
 gamma.fun - function(mu,sd,start=100)
 {
 f.fn - function(alpha)

sd^2-mu^2/(gamma(1+1/alpha))^2*(gamma(1+2/alpha)-(gamma(1+1/alpha))^2)
 alpha - optim(start, f.fn,method='BFGS')
 beta - mu/gamma(1+1/alpha$par)
 return(list=c(a=alpha$par,b=beta));
 }
 
 
 But the problems come up here:
 
 1)  the return values of a and b are only related to
 the input mean, and nothing to do with the sd. For
 instance, when I apply a mean mu = 3 whatever I use
 sd=2, sd=4, the function returned the same scale and
 shape values.
 
  gamma.fun(3,4,10);
ab 
 5.112554 3.263178 
 
  gamma.fun(3,2,10);
ab 
 5.112554 3.263178 
 
 2) the start value determines the results: if I
 apply mean = 3, and sd=2, with a start of 10, it
 would return alpha close to 10, if I use a start =
 100, it would return alpha close to 100.
 
  gamma.fun(3,2,10);
ab 
 5.112554 3.263178 
 
  gamma.fun(3,2,100);
 a b 
 99.71  3.017120 
 
 Since I am not a statistician, I guess there must be
 some theoretical reasons wrong with this question.
 So I am looking forward to some correction and
 advice to solve these. Thanks a lot in advance!
 
 Leaf
 
   [[alternative HTML version deleted]]
 
 __
 R-help@stat.math.ethz.ch mailing list
 https://stat.ethz.ch/mailman/listinfo/r-help
 PLEASE do read the posting guide!
 http://www.R-project.org/posting-guide.html


__
R-help@stat.math.ethz.ch mailing list
https://stat.ethz.ch/mailman/listinfo/r-help
PLEASE do read the posting guide http://www.R-project.org/posting-guide.html
and provide commented, minimal, self-contained, reproducible code.


Re: [R] Order-restricted inference

2006-07-21 Thread Thomas Lumley
On Thu, 20 Jul 2006, Bruno L. Giordano wrote:

 If R packages are not found for my purpose, can somebody please point me to
 some recent monographs on order-restricted inference?


One more recent book is:
   Robertson, T., Wright, F.,  Dykstra, R. (1988). Order restricted
   statistical inference. New York: John Wiley and Sons.

-thomas

Thomas Lumley   Assoc. Professor, Biostatistics
[EMAIL PROTECTED]   University of Washington, Seattle

__
R-help@stat.math.ethz.ch mailing list
https://stat.ethz.ch/mailman/listinfo/r-help
PLEASE do read the posting guide http://www.R-project.org/posting-guide.html
and provide commented, minimal, self-contained, reproducible code.


Re: [R] positive semi-definite matrix

2006-07-21 Thread Francisco J. Zagmutt
Take a look at make.positive.definite in the corpcor package.  The 
implementation is very similar to what Duncan suggested.

Regards,

Francisco

Dr. Francisco J. Zagmutt
College of Veterinary Medicine and Biomedical Sciences
Colorado State University




From: Duncan Murdoch [EMAIL PROTECTED]
To: roger bos [EMAIL PROTECTED]
CC: RHelp r-help@stat.math.ethz.ch
Subject: Re: [R] positive semi-definite matrix
Date: Fri, 21 Jul 2006 09:44:42 -0400

On 7/21/2006 8:59 AM, roger bos wrote:
  I have a covariance matrix that is not positive semi-definite matrix and 
I
  need it to be via some sort of adjustment.  Is there any R routine or
  package to help me do this?

I think you need to be more specific about what have and what you want,
but if the matrix is symmetric and nearly positive semi-definite (but
not exactly because of rounding error), you might try something like

fixit - function(A) {
eig - eigen(A, symmetric = TRUE)
eig$values - pmax(0, eig$values)
return(eig$vectors %*% diag(eig$values) %*% t(eig$vectors))
}

Rounding error means this is not guaranteed to be positive
semi-definite, but it will be very close.

Duncan Murdoch

__
R-help@stat.math.ethz.ch mailing list
https://stat.ethz.ch/mailman/listinfo/r-help
PLEASE do read the posting guide 
http://www.R-project.org/posting-guide.html
and provide commented, minimal, self-contained, reproducible code.

__
R-help@stat.math.ethz.ch mailing list
https://stat.ethz.ch/mailman/listinfo/r-help
PLEASE do read the posting guide http://www.R-project.org/posting-guide.html
and provide commented, minimal, self-contained, reproducible code.


Re: [R] positive semi-definite matrix

2006-07-21 Thread Ravi Varadhan
There is a paper by N.J. Higham (SIAM J Matrix Anal, 1998) on a modified
cholesky decomposition of symmetric and not necessarily positive definite
matrix (say, A), with an important goal of producing a small-normed
perturbation of A (say, delA), that makes (A + delA) positive definite.
http://epubs.siam.org/sam-bin/dbq/article/30289

There is also an algorithm in Gill, Murray and Wright's text - Practical
Optimization (section 4.4.2).

These may be relevant to your problem.  I am not sure if these algorithms
have been implemented in R, for example, in the matrix library. 

Ravi.


--
Ravi Varadhan, Ph.D.
Assistant Professor,  The Center on Aging and Health
Division of Geriatric Medicine and Gerontology
Johns Hopkins University
Ph: (410) 502-2619
Fax: (410) 614-9625
Email:  [EMAIL PROTECTED]
Webpage: http://www.jhsph.edu/agingandhealth/People/Faculty/Varadhan.html 
--

 -Original Message-
 From: [EMAIL PROTECTED] [mailto:r-help-
 [EMAIL PROTECTED] On Behalf Of Duncan Murdoch
 Sent: Friday, July 21, 2006 9:45 AM
 To: roger bos
 Cc: RHelp
 Subject: Re: [R] positive semi-definite matrix
 
 On 7/21/2006 8:59 AM, roger bos wrote:
  I have a covariance matrix that is not positive semi-definite matrix and
 I
  need it to be via some sort of adjustment.  Is there any R routine or
  package to help me do this?
 
 I think you need to be more specific about what have and what you want,
 but if the matrix is symmetric and nearly positive semi-definite (but
 not exactly because of rounding error), you might try something like
 
 fixit - function(A) {
eig - eigen(A, symmetric = TRUE)
eig$values - pmax(0, eig$values)
return(eig$vectors %*% diag(eig$values) %*% t(eig$vectors))
 }
 
 Rounding error means this is not guaranteed to be positive
 semi-definite, but it will be very close.
 
 Duncan Murdoch
 
 __
 R-help@stat.math.ethz.ch mailing list
 https://stat.ethz.ch/mailman/listinfo/r-help
 PLEASE do read the posting guide http://www.R-project.org/posting-
 guide.html
 and provide commented, minimal, self-contained, reproducible code.

__
R-help@stat.math.ethz.ch mailing list
https://stat.ethz.ch/mailman/listinfo/r-help
PLEASE do read the posting guide http://www.R-project.org/posting-guide.html
and provide commented, minimal, self-contained, reproducible code.


Re: [R] Weibull distribution

2006-07-21 Thread Thomas Lumley
On Fri, 21 Jul 2006, Valentin Dimitrov wrote:

 Dear Leaf,

 I modified your code as follows:

 gamma.fun - function(mu,sd,start=100)
 {
 f.fn - function(alpha)
 {abs(sd^2-mu^2/(gamma(1+1/alpha))^2*(gamma(1+2/alpha)-(gamma(1+1/alpha))^2))}
 alpha - optim(start, f.fn)
 beta - mu/gamma(1+1/alpha$par)
 return(list=c(a=alpha$par,b=beta));
 }

 Now it works properly.

 First, I added an abs(). You tried to solve an
 equation by means of the R-function optim(), which
 finds a minimum. That's why you can find the solution
 of f(x)=a through minimization of abs(f(x)-a).
 Second, I deleted the optim-method BFGS from the
 optim() function, because it is not appropriate in
 this case.

optim() is not appropriate at all in this case -- its help page says to 
use optimize() for one-dimensional problems.

In fact, in one dimension there isn't any need to resort to optimization 
when you really want root-finding, and uniroot() is more appropriate than 
optimize().


-thomas

__
R-help@stat.math.ethz.ch mailing list
https://stat.ethz.ch/mailman/listinfo/r-help
PLEASE do read the posting guide http://www.R-project.org/posting-guide.html
and provide commented, minimal, self-contained, reproducible code.


[R] Problems with character spacing in windows metafiles...

2006-07-21 Thread Dan Linden

This problem was posted a couple years ago here:

http://tolstoy.newcastle.edu.au/R/help/04/01/0231.html

Using Windows XP with R 2.3.1, I am now experiencing the same problem again:
when a plot is saved and/or copied as a WMF, the labels do not have the
correct character spacing.  I am trying to insert the WMF into a MS Word
2003 document (my first mistake, I know), but even when the WMF is opened
with other graphics software, the problem remains.  Must be something with
the way the WMF is written.

The strange this is that I was able to do this on another computer with a
slilghtly older version of R (2.2.x?), and did not have the problem.  The
version of Microsoft Word was the same on both, and the code did not matter
(any basic plot with any font had its text labels squeezed).

I've done some searching and haven't come up with any good solutions.  For
Windows users, the WMF is the most convenient format for saving vector
graphics.  EPS is not an option for me.

Any help would be greatly appreciated.

Cheers,
Dan
-- 
View this message in context: 
http://www.nabble.com/Problems-with-character-spacing-in-windows-metafiles...-tf1980921.html#a5435945
Sent from the R help forum at Nabble.com.

__
R-help@stat.math.ethz.ch mailing list
https://stat.ethz.ch/mailman/listinfo/r-help
PLEASE do read the posting guide http://www.R-project.org/posting-guide.html
and provide commented, minimal, self-contained, reproducible code.


Re: [R] : Arial font for text in lattice plots under Linux

2006-07-21 Thread Prof Brian Ripley
This is a question about devices, not lattice per se, so applies to all 
forms of plotting.

For jpeg, you can specify the fonts via the 'fonts' argument: see ?X11.

For postscript, you can convert the fonts to Type 1 via e.g. ttf2pt1, or 
you can use ttf2afm to make .afm files and a postscript interpreter that 
can handle TrueType fonts.

On Fri, 21 Jul 2006, Peter Ho wrote:

 Hi,
 
 I have been asked by a publisher to change the font style of a lattice 
 plot in my manuscript. I have consulted documentation on trellis 
 graphics and the excellent book R graphics, but am still not sure how 
 I could create plots with Arial as the font style for text in the plot.  
 I am running R (Version 2.3.1 (2006-06-01)) under debian Linux. I have 
 msttcorefonts installed.
 
 Will it be possible to do this with jpeg() as a device? Or with 
 postscript()?
 
 
 
 
 Peter
 
 __
 R-help@stat.math.ethz.ch mailing list
 https://stat.ethz.ch/mailman/listinfo/r-help
 PLEASE do read the posting guide http://www.R-project.org/posting-guide.html
 and provide commented, minimal, self-contained, reproducible code.
 

-- 
Brian D. Ripley,  [EMAIL PROTECTED]
Professor of Applied Statistics,  http://www.stats.ox.ac.uk/~ripley/
University of Oxford, Tel:  +44 1865 272861 (self)
1 South Parks Road, +44 1865 272866 (PA)
Oxford OX1 3TG, UKFax:  +44 1865 272595

__
R-help@stat.math.ethz.ch mailing list
https://stat.ethz.ch/mailman/listinfo/r-help
PLEASE do read the posting guide http://www.R-project.org/posting-guide.html
and provide commented, minimal, self-contained, reproducible code.


Re: [R] Reading data with blank elements

2006-07-21 Thread Marc Schwartz (via MN)
On Fri, 2006-07-21 at 05:43 -0700, Ahamarshan jn wrote:
 Hi,
  I have a dataset saved in *.csv format, that contains
 13 columns (the first column being the title name and
 the rest experiments) and about 2500 rows.
 Not all columns in the row have data in it
 i.e for eg
   
 BS00,-0.084,0.0136,-0.1569,-0.6484,1.103,1.7859,0.40287,0.5368,0.08461,-0.1935,-0.147974,0.30685
 
 BS01,0.491270283,0.875826172,,
 
 BS02,0.090794476,0.225858954,,,0.32643,0.34317,0.133145295,,,0.115832599,0.47636458,
 
 BS03,0.019828221,-0.095735935,-0.122767219,-0.0676,0.002533,-0.1510361,0.736247,2.053192,-0.423658,0.4591219,1.1245015,
 
 BS04,-0.435189342,-0.041595955,-0.781281128,-1.923036,-3.2301671020.152322609,-1.495513519,,
   
 
 I am using R to perform a correlation, but I am
 getting an error while trying to read the data as
 
 
 
 person.data-read.table(datafile.csv,header=TRUE,sep=',',row.names=1)
 
 Error in scan (file = file, what = what, sep = sep,
 quote = quote, dec = dec,  : 
 line 1919 did not have 13 elements
 Execution halted 
  
 The error looks as though there is a problem with the
 last element being not read when it is blank. I could
 introduce terms like na to the blank elements but I
 donot want to do that because this will hinder my
 future analysis. 
 
 Can some one suggest me a solution to overcome this
 problem while reading the data? , or is there
 something that I have missed to make the data
 readable. 
 
 Thank you in advance, 
 
 PS: The data was imported from a experiment and saved
 in excel sheet as a *.csv and then used.

You have already had other replies, to which I would add, be sure to
read Chapter 8 in the R Import/Export Manual regarding importing Excel
files and other options besides exporting to a CSV file.

In addition, the issue of Excel generating CSV files with the last
column missing on some rows is a known issue and is reported in the MSKB
here:

http://support.microsoft.com/default.aspx?scid=kb;EN-US;q77295

Even though the latest version of Excel listed in the article as being
relevant is 97, I had this problem with 2000 and 2003 as well.

I would instead use OpenOffice.org's Calc to do the export when this was
required. Calc did not have this problem.

HTH,

Marc Schwartz

__
R-help@stat.math.ethz.ch mailing list
https://stat.ethz.ch/mailman/listinfo/r-help
PLEASE do read the posting guide http://www.R-project.org/posting-guide.html
and provide commented, minimal, self-contained, reproducible code.


[R] Multcomp plotting

2006-07-21 Thread Nair, Murlidharan T
I am using the multcomp package for doing multiple comparisons. Since the data 
I am handling is huge the number of comparisons are also large. I am interested 
in:
1  Breaking down my plots to get rid of the clutter that happens when plotting 
the entire data set. How do I pass only part of the data to the plot function ?
 
fungus.cirec-simint(Fungus.yield~Habitat, data=fungus,conf.level=0.95,type 
=c(Tukey))
plot(fungus.cirec)  #This plots the entire data. I want to plot part of the 
data only
 
2I am also interested in getting rid of the field name associated with each 
categorical variable. 
Here is what the part of the data looks like
 
Habitat Fungus.yield
Birch 20.83829053
Birch 22.9718181
Birch 22.28216829
Birch 24.23136797
Birch 22.32147961
Birch 20.30783598
Oak 27.24047258
Oak 29.7730014
Oak 30.12608508
Oak 25.76088669
Oak 30.14750974
Hornbeam 17.05307949
Hornbeam 15.32805111
Hornbeam 18.26920177
Hornbeam 21.30987049
Hornbeam 21.7173223
When it plots labels it as HabitatBirch-HabitatOak for example.  How do I get 
rid of the field name Habitat in the plot?
 
3 How do I tell the method to mark the significant comparisons? (i.e those 
that do not intersect the zero line).
 
Thanks ../Murli

__
R-help@stat.math.ethz.ch mailing list
https://stat.ethz.ch/mailman/listinfo/r-help
PLEASE do read the posting guide http://www.R-project.org/posting-guide.html
and provide commented, minimal, self-contained, reproducible code.


[R] seeking robust test for equality of variances w/ observation weights

2006-07-21 Thread Alexis Diamond
Hello R community,

I am looking for a robust test for equality of variances that can take
observation weights.
I realize I can do the F-test with weighted variances, but I've read that
this test is not very robust.

So I thought about maybe adding a weights argument to John Fox's code for
the Levene Test (in the car library, levene.test),
substituting his median function for a  weighted.mean and also including
the observation weights in his lm run--
after all, Levene's original test used the mean, not the median.

I asked John about it and he doesn't know what the properties of this
weighted Levene test would be.
Does anyone have any thoughts or suggestions, or know of a robust weighted
hypothesis test for equality of variances?

Thank you in advance for any advice you can provide,

Alexis Diamond
[EMAIL PROTECTED]

[[alternative HTML version deleted]]

__
R-help@stat.math.ethz.ch mailing list
https://stat.ethz.ch/mailman/listinfo/r-help
PLEASE do read the posting guide http://www.R-project.org/posting-guide.html
and provide commented, minimal, self-contained, reproducible code.


[R] multicomp plotting

2006-07-21 Thread Nair, Murlidharan T

I am using the multcomp package for doing multiple comparisons. Since
the data I am handling is huge the number of comparisons are also large.
I am interested in:
1  Breaking down my plots to get rid of the clutter that happens when
plotting the entire data set. How do I pass only part of the data to the
plot function ?
 
fungus.cirec-simint(Fungus.yield~Habitat,
data=fungus,conf.level=0.95,type =c(Tukey))
plot(fungus.cirec)  #This plots the entire data. I want to plot part of
the data only
 
2I am also interested in getting rid of the field name associated with
each categorical variable. 
Here is what the part of the data looks like
 
Habitat Fungus.yield
Birch 20.83829053
Birch 22.9718181
Birch 22.28216829
Birch 24.23136797
Birch 22.32147961
Birch 20.30783598
Oak 27.24047258
Oak 29.7730014
Oak 30.12608508
Oak 25.76088669
Oak 30.14750974
Hornbeam 17.05307949
Hornbeam 15.32805111
Hornbeam 18.26920177
Hornbeam 21.30987049
Hornbeam 21.7173223
It plots labels as HabitatBirch-HabitatOak for example.  How do I get
rid of the field name Habitat in the plot? 
 
3 How do I tell the method to mark the significant comparisons? (i.e
those that do not intersect the zero line).
 
Thanks ../Murli

__
R-help@stat.math.ethz.ch mailing list
https://stat.ethz.ch/mailman/listinfo/r-help
PLEASE do read the posting guide http://www.R-project.org/posting-guide.html
and provide commented, minimal, self-contained, reproducible code.


[R] A statistical question

2006-07-21 Thread Bi-Info (http://members.home.nl/bi-info)
Dear Users,

I have two particular problems that I need to solve.

I do an analysis of a survey about sexuality.

(Don't read any further if you don't like the subject.)

Problem (1)

In the survey there a two questions about monogamy.

(1) Are you monogamous? (At this moment.)
(2) Have you been in contact with men and / or women? (Past / Present)

For some other inferences I need to extract historical data out of these 
questions about monogamy, like past monogamy (which hasn't been asked). 
This should be possible. Is there a reasonable way out of here?

Problem (2)

I have to do some geographical testing. I have to check the geographical 
distribution of respondents and some other properties. Could you advise 
me what to read / use? An R package with help is sufficient (I hope).


Thanks,

Wilfred

__
R-help@stat.math.ethz.ch mailing list
https://stat.ethz.ch/mailman/listinfo/r-help
PLEASE do read the posting guide http://www.R-project.org/posting-guide.html
and provide commented, minimal, self-contained, reproducible code.


Re: [R] positive semi-definite matrix

2006-07-21 Thread Martin Maechler
 Ravi == Ravi Varadhan [EMAIL PROTECTED]
 on Fri, 21 Jul 2006 11:33:23 -0400 writes:

Ravi There is a paper by N.J. Higham (SIAM J Matrix Anal,
Ravi 1998) on a modified cholesky decomposition of
Ravi symmetric and not necessarily positive definite matrix
Ravi (say, A), with an important goal of producing a
Ravi small-normed perturbation of A (say, delA), that
Ravi makes (A + delA) positive definite.

Ravi http://epubs.siam.org/sam-bin/dbq/article/30289

Ravi There is also an algorithm in Gill, Murray and
Ravi Wright's text - Practical Optimization (section
Ravi 4.4.2).

Thanks a lot, Ravi,
for the interesting references, in the past I once had looked
for such things but did not find any --- most probably because I
used wrong keywords.

Ravi These may be relevant to your problem.  I am not sure
Ravi if these algorithms have been implemented in R, for
Ravi example, in the matrix library.

O... !  It's  Matrix and  `package', yes `package', yes `package' ..

but no, it hasn't been implemented there yet, AFAIK.
OTOH, it's not a bad idea to do there, since it's building on
the  LDL' cholesky factorization   which we are using
in Matrix in other places anyway.

Thanks again for your help!
Martin Maechler, ETH Zurich

Ravi 
--
Ravi Ravi Varadhan, Ph.D.
Ravi Assistant Professor,  The Center on Aging and Health
Ravi Division of Geriatric Medicine and Gerontology
Ravi Johns Hopkins University
Ravi Ph: (410) 502-2619
Ravi Fax: (410) 614-9625
Ravi Email:  [EMAIL PROTECTED]
Ravi Webpage: 
http://www.jhsph.edu/agingandhealth/People/Faculty/Varadhan.html 
Ravi 
--

 -Original Message-
 From: [EMAIL PROTECTED] [mailto:r-help-
 [EMAIL PROTECTED] On Behalf Of Duncan Murdoch
 Sent: Friday, July 21, 2006 9:45 AM
 To: roger bos
 Cc: RHelp
 Subject: Re: [R] positive semi-definite matrix
 
 On 7/21/2006 8:59 AM, roger bos wrote:
  I have a covariance matrix that is not positive semi-definite matrix 
and
 I
  need it to be via some sort of adjustment.  Is there any R routine or
  package to help me do this?
 
 I think you need to be more specific about what have and what you want,
 but if the matrix is symmetric and nearly positive semi-definite (but
 not exactly because of rounding error), you might try something like
 
 fixit - function(A) {
 eig - eigen(A, symmetric = TRUE)
 eig$values - pmax(0, eig$values)
 return(eig$vectors %*% diag(eig$values) %*% t(eig$vectors))
 }
 
 Rounding error means this is not guaranteed to be positive
 semi-definite, but it will be very close.
 
 Duncan Murdoch

__
R-help@stat.math.ethz.ch mailing list
https://stat.ethz.ch/mailman/listinfo/r-help
PLEASE do read the posting guide http://www.R-project.org/posting-guide.html
and provide commented, minimal, self-contained, reproducible code.


Re: [R] Order-restricted inference

2006-07-21 Thread Bruno L. Giordano
I found this other more recent monograph:

Silvapulle and Sen (October 2004), Constrained Statistical Inference: Order,
Inequality, and Shape Constraints, Wiley-Interscience.

http://ca.wiley.com/WileyCDA/WileyTitle/productCd-0471208272,descCd-reviews.html

Thanks for the tip,
Bruno


- Original Message - 
From: Thomas Lumley [EMAIL PROTECTED]
To: Bruno L. Giordano [EMAIL PROTECTED]
Cc: r-help@stat.math.ethz.ch
Sent: Friday, July 21, 2006 11:22 AM
Subject: Re: [R] Order-restricted inference


 On Thu, 20 Jul 2006, Bruno L. Giordano wrote:

 If R packages are not found for my purpose, can somebody please point me
 to
 some recent monographs on order-restricted inference?


 One more recent book is:
   Robertson, T., Wright, F.,  Dykstra, R. (1988). Order restricted
   statistical inference. New York: John Wiley and Sons.

  -thomas

 Thomas Lumley Assoc. Professor, Biostatistics
 [EMAIL PROTECTED] University of Washington, Seattle


~~
Bruno L. Giordano, Ph.D.
CIRMMT
Schulich School of Music, McGill University
555 Sherbrooke Street West
Montréal, QC H3A 1E3
Canada
http://www.music.mcgill.ca/~bruno/

__
R-help@stat.math.ethz.ch mailing list
https://stat.ethz.ch/mailman/listinfo/r-help
PLEASE do read the posting guide http://www.R-project.org/posting-guide.html
and provide commented, minimal, self-contained, reproducible code.


Re: [R] seeking robust test for equality of variances w/ observationweights

2006-07-21 Thread Berton Gunter
You can always bootstrap any robust spread measure (e.g. mad or higher
efficiency versions from robustbase or other packages).

-- Bert Gunter
Genentech Non-Clinical Statistics
South San Francisco, CA
 
The business of the statistician is to catalyze the scientific learning
process.  - George E. P. Box
 
 

 -Original Message-
 From: [EMAIL PROTECTED] 
 [mailto:[EMAIL PROTECTED] On Behalf Of Alexis Diamond
 Sent: Friday, July 21, 2006 9:56 AM
 To: r-help@stat.math.ethz.ch
 Subject: [R] seeking robust test for equality of variances w/ 
 observationweights
 
 Hello R community,
 
 I am looking for a robust test for equality of variances that can take
 observation weights.
 I realize I can do the F-test with weighted variances, but 
 I've read that
 this test is not very robust.
 
 So I thought about maybe adding a weights argument to John 
 Fox's code for
 the Levene Test (in the car library, levene.test),
 substituting his median function for a  weighted.mean and 
 also including
 the observation weights in his lm run--
 after all, Levene's original test used the mean, not the median.
 
 I asked John about it and he doesn't know what the properties of this
 weighted Levene test would be.
 Does anyone have any thoughts or suggestions, or know of a 
 robust weighted
 hypothesis test for equality of variances?
 
 Thank you in advance for any advice you can provide,
 
 Alexis Diamond
 [EMAIL PROTECTED]
 
   [[alternative HTML version deleted]]
 
 __
 R-help@stat.math.ethz.ch mailing list
 https://stat.ethz.ch/mailman/listinfo/r-help
 PLEASE do read the posting guide 
 http://www.R-project.org/posting-guide.html
 and provide commented, minimal, self-contained, reproducible code.


__
R-help@stat.math.ethz.ch mailing list
https://stat.ethz.ch/mailman/listinfo/r-help
PLEASE do read the posting guide http://www.R-project.org/posting-guide.html
and provide commented, minimal, self-contained, reproducible code.


[R] intersect of list elements

2006-07-21 Thread Georg Otto

Hi,

i have a list of several vectors, for example:

 vectorlist
$vector.a.1
[1] a b c

$vector.a.2
[1] a b d

$vector.b.1
[1] e f g


I can use intersect to find elements that appear in $vector.a.1 and
$vector.a.2:

 intersect(vectorlist[[1]], vectorlist[[2]])
[1] a b


I would like to use grep to get the vectors by their names matching an
expression and to find the intersects between those vectors. For the
first step:

 vectorlist[grep (vector.a, names(vectorlist))]
$vector.a.1
[1] a b c

$vector.a.2
[1] a b d


Unfortunately, I can not pass the two vectors as argument to intersect:

 intersect(vectorlist[grep (vector.a, names(vectorlist))])
Error in unique(y[match(x, y, 0)]) : argument y is missing, with no default

I am running R Version 2.3.1 (2006-06-01) 


Could somone help me to solve this?

Cheers,

Georg

__
R-help@stat.math.ethz.ch mailing list
https://stat.ethz.ch/mailman/listinfo/r-help
PLEASE do read the posting guide http://www.R-project.org/posting-guide.html
and provide commented, minimal, self-contained, reproducible code.


Re: [R] intersect of list elements

2006-07-21 Thread Berton Gunter
FAQ 7.21.

But there are perhaps slicker ways.

-- Bert Gunter
Genentech Non-Clinical Statistics
South San Francisco, CA
 
 

 -Original Message-
 From: [EMAIL PROTECTED] 
 [mailto:[EMAIL PROTECTED] On Behalf Of Georg Otto
 Sent: Friday, July 21, 2006 10:39 AM
 To: r-help@stat.math.ethz.ch
 Subject: [R] intersect of list elements
 
 
 Hi,
 
 i have a list of several vectors, for example:
 
  vectorlist
 $vector.a.1
 [1] a b c
 
 $vector.a.2
 [1] a b d
 
 $vector.b.1
 [1] e f g
 
 
 I can use intersect to find elements that appear in $vector.a.1 and
 $vector.a.2:
 
  intersect(vectorlist[[1]], vectorlist[[2]])
 [1] a b
 
 
 I would like to use grep to get the vectors by their names matching an
 expression and to find the intersects between those vectors. For the
 first step:
 
  vectorlist[grep (vector.a, names(vectorlist))]
 $vector.a.1
 [1] a b c
 
 $vector.a.2
 [1] a b d
 
 
 Unfortunately, I can not pass the two vectors as argument to 
 intersect:
 
  intersect(vectorlist[grep (vector.a, names(vectorlist))])
 Error in unique(y[match(x, y, 0)]) : argument y is missing, 
 with no default
 
 I am running R Version 2.3.1 (2006-06-01) 
 
 
 Could somone help me to solve this?
 
 Cheers,
 
 Georg
 
 __
 R-help@stat.math.ethz.ch mailing list
 https://stat.ethz.ch/mailman/listinfo/r-help
 PLEASE do read the posting guide 
 http://www.R-project.org/posting-guide.html
 and provide commented, minimal, self-contained, reproducible code.


__
R-help@stat.math.ethz.ch mailing list
https://stat.ethz.ch/mailman/listinfo/r-help
PLEASE do read the posting guide http://www.R-project.org/posting-guide.html
and provide commented, minimal, self-contained, reproducible code.


Re: [R] intersect of list elements

2006-07-21 Thread Sundar Dorai-Raj


Georg Otto wrote:
 Hi,
 
 i have a list of several vectors, for example:
 
 
vectorlist
 
 $vector.a.1
 [1] a b c
 
 $vector.a.2
 [1] a b d
 
 $vector.b.1
 [1] e f g
 
 
 I can use intersect to find elements that appear in $vector.a.1 and
 $vector.a.2:
 
 
intersect(vectorlist[[1]], vectorlist[[2]])
 
 [1] a b
 
 
 I would like to use grep to get the vectors by their names matching an
 expression and to find the intersects between those vectors. For the
 first step:
 
 
vectorlist[grep (vector.a, names(vectorlist))]
 
 $vector.a.1
 [1] a b c
 
 $vector.a.2
 [1] a b d
 
 
 Unfortunately, I can not pass the two vectors as argument to intersect:
 
 
intersect(vectorlist[grep (vector.a, names(vectorlist))])
 
 Error in unique(y[match(x, y, 0)]) : argument y is missing, with no default
 
 I am running R Version 2.3.1 (2006-06-01) 
 
 
 Could somone help me to solve this?
 
 Cheers,
 
 Georg
 
 __
 R-help@stat.math.ethz.ch mailing list
 https://stat.ethz.ch/mailman/listinfo/r-help
 PLEASE do read the posting guide http://www.R-project.org/posting-guide.html
 and provide commented, minimal, self-contained, reproducible code.


Will this work for you?

vectorlist - list(vector.a.1 = c(a, b, c),
vector.a.2 = c(a, b, d),
vector.b.1 = c(e, f, g))

intersect2 - function(...) {
   args - list(...)
   nargs - length(args)
   if(nargs = 1) {
 if(nargs == 1  is.list(args[[1]])) {
   do.call(intersect2, args[[1]])
 } else {
   stop(cannot evaluate intersection fewer than 2 arguments)
 }
   } else if(nargs == 2) {
 intersect(args[[1]], args[[2]])
   } else {
 intersect(args[[1]], intersect2(args[-1]))
   }
}

vector.a - vectorlist[grep (vector.a, names(vectorlist))]
intersect2(vector.a)
intersect2(vectorlist)

HTH,

--sundar

__
R-help@stat.math.ethz.ch mailing list
https://stat.ethz.ch/mailman/listinfo/r-help
PLEASE do read the posting guide http://www.R-project.org/posting-guide.html
and provide commented, minimal, self-contained, reproducible code.


Re: [R] intersect of list elements

2006-07-21 Thread Gabor Grothendieck
The following assumes that within each component of vectorlist
the vector elements are unique. In that case the first two lines
define vectorlist and perform the grep, as in your post.  Elements
of the intersection must occur n times where n is the number
of components of vectorlist that match the grep and those
elements are extracted in the last line.

# from your post
vectorlist - list(vector.a.1 = c(a, b, c), vector.a.2 = c(a,
   b, d), vector.b.1. = c(e, f, g))
idx - grep(vector.a, names(vectorlist))

# get intersection
names(which(table(unlist(vectorlist[idx])) == length(idx)))

On 7/21/06, Georg Otto [EMAIL PROTECTED] wrote:

 Hi,

 i have a list of several vectors, for example:

  vectorlist
 $vector.a.1
 [1] a b c

 $vector.a.2
 [1] a b d

 $vector.b.1
 [1] e f g


 I can use intersect to find elements that appear in $vector.a.1 and
 $vector.a.2:

  intersect(vectorlist[[1]], vectorlist[[2]])
 [1] a b


 I would like to use grep to get the vectors by their names matching an
 expression and to find the intersects between those vectors. For the
 first step:

  vectorlist[grep (vector.a, names(vectorlist))]
 $vector.a.1
 [1] a b c

 $vector.a.2
 [1] a b d


 Unfortunately, I can not pass the two vectors as argument to intersect:

  intersect(vectorlist[grep (vector.a, names(vectorlist))])
 Error in unique(y[match(x, y, 0)]) : argument y is missing, with no default

 I am running R Version 2.3.1 (2006-06-01)


 Could somone help me to solve this?

 Cheers,

 Georg

 __
 R-help@stat.math.ethz.ch mailing list
 https://stat.ethz.ch/mailman/listinfo/r-help
 PLEASE do read the posting guide http://www.R-project.org/posting-guide.html
 and provide commented, minimal, self-contained, reproducible code.


__
R-help@stat.math.ethz.ch mailing list
https://stat.ethz.ch/mailman/listinfo/r-help
PLEASE do read the posting guide http://www.R-project.org/posting-guide.html
and provide commented, minimal, self-contained, reproducible code.


[R] connection to X11 problem

2006-07-21 Thread Paquet, Agnes
Dear List,

I am a new Mac user and I am having problem generating png (or jpeg)
using the GUI version of R. I installed R-2.3.1.dmg (custom install with
everything selected) and X11User.pkg but I am still getting the
following X11 connection error when I try to generate a png (or a jpeg):

Error in X11(paste(png::, filename, sep = ), width, height,
pointsize,  : 
unable to start device PNG
In addition: Warning message:
unable to open connection to X11 display ''

I tried to set up the DISPLAY variable using the command:

Sys.putenv(DISPLAY=:0)

but I am still running into the same problem. 

Is there anything else I need to do or install in order to use X11? I am
using a intel Core Duo processor and OSX 10.4.7.

Thank you for your help,

Agnes

__
R-help@stat.math.ethz.ch mailing list
https://stat.ethz.ch/mailman/listinfo/r-help
PLEASE do read the posting guide http://www.R-project.org/posting-guide.html
and provide commented, minimal, self-contained, reproducible code.


Re: [R] Problems with character spacing in windows metafiles...

2006-07-21 Thread Dan Linden

I finally determined the problem, but not the specific cause.

For some reason, the WMF is resized when saved or copied from R.  If you
compare the same plot pasted into PowerPoint as both a bitmap and a WMF, the
WMF has its height increased ~14% and its width decreased ~5% consistently. 
This is true whether you copy/paste or save the WMF.

At least, that is what occurs on my Windows XP machine with Office 2003 and
R 2.3.1 installed.
-- 
View this message in context: 
http://www.nabble.com/Problems-with-character-spacing-in-windows-metafiles...-tf1980921.html#a5438500
Sent from the R help forum at Nabble.com.

__
R-help@stat.math.ethz.ch mailing list
https://stat.ethz.ch/mailman/listinfo/r-help
PLEASE do read the posting guide http://www.R-project.org/posting-guide.html
and provide commented, minimal, self-contained, reproducible code.


Re: [R] connection to X11 problem

2006-07-21 Thread Marc Schwartz (via MN)
On Fri, 2006-07-21 at 11:47 -0700, Paquet, Agnes wrote:
 Dear List,
 
 I am a new Mac user and I am having problem generating png (or jpeg)
 using the GUI version of R. I installed R-2.3.1.dmg (custom install with
 everything selected) and X11User.pkg but I am still getting the
 following X11 connection error when I try to generate a png (or a jpeg):
 
 Error in X11(paste(png::, filename, sep = ), width, height,
 pointsize,  : 
 unable to start device PNG
 In addition: Warning message:
 unable to open connection to X11 display ''
 
 I tried to set up the DISPLAY variable using the command:
 
 Sys.putenv(DISPLAY=:0)
 
 but I am still running into the same problem. 
 
 Is there anything else I need to do or install in order to use X11? I am
 using a intel Core Duo processor and OSX 10.4.7.
 
 Thank you for your help,
 
 Agnes

I don't use a Mac, but the following might be helpful:

http://cran.r-project.org/bin/macosx/RMacOSX-FAQ.html#X11-window-server-_0028optional_0029

http://finzi.psych.upenn.edu/R/Rhelp02a/archive/77424.html

http://finzi.psych.upenn.edu/R/Rhelp02a/archive/46097.html


Also note that there is a R-sig-Mac e-mail list:

https://stat.ethz.ch/mailman/listinfo/r-sig-mac


HTH,

Marc Schwartz

__
R-help@stat.math.ethz.ch mailing list
https://stat.ethz.ch/mailman/listinfo/r-help
PLEASE do read the posting guide http://www.R-project.org/posting-guide.html
and provide commented, minimal, self-contained, reproducible code.


Re: [R] positive semi-definite matrix

2006-07-21 Thread Ravi Varadhan
Martin,

You are most welcome.  I apologize for my faux pas.  I really did mean to
say Matrix package, but got sloppy!

There is also another (more recent) article by Higham:
http://www.maths.man.ac.uk/~nareports/narep369.pdf
 

Best,
Ravi.

--
Ravi Varadhan, Ph.D.
Assistant Professor,  The Center on Aging and Health
Division of Geriatric Medicine and Gerontology
Johns Hopkins University
Ph: (410) 502-2619
Fax: (410) 614-9625
Email:  [EMAIL PROTECTED]
Webpage: http://www.jhsph.edu/agingandhealth/People/Faculty/Varadhan.html 
--
 -Original Message-
 From: Martin Maechler [mailto:[EMAIL PROTECTED]
 Sent: Friday, July 21, 2006 1:14 PM
 To: Ravi Varadhan
 Cc: 'Duncan Murdoch'; 'roger bos'; 'RHelp'; [EMAIL PROTECTED]
 Subject: Re: [R] positive semi-definite matrix
 
  Ravi == Ravi Varadhan [EMAIL PROTECTED]
  on Fri, 21 Jul 2006 11:33:23 -0400 writes:
 
 Ravi There is a paper by N.J. Higham (SIAM J Matrix Anal,
 Ravi 1998) on a modified cholesky decomposition of
 Ravi symmetric and not necessarily positive definite matrix
 Ravi (say, A), with an important goal of producing a
 Ravi small-normed perturbation of A (say, delA), that
 Ravi makes (A + delA) positive definite.
 
 Ravi http://epubs.siam.org/sam-bin/dbq/article/30289
 
 Ravi There is also an algorithm in Gill, Murray and
 Ravi Wright's text - Practical Optimization (section
 Ravi 4.4.2).
 
 Thanks a lot, Ravi,
 for the interesting references, in the past I once had looked
 for such things but did not find any --- most probably because I
 used wrong keywords.
 
 Ravi These may be relevant to your problem.  I am not sure
 Ravi if these algorithms have been implemented in R, for
 Ravi example, in the matrix library.
 
 O... !  It's  Matrix and  `package', yes `package', yes `package' ..
 
 but no, it hasn't been implemented there yet, AFAIK.
 OTOH, it's not a bad idea to do there, since it's building on
 the  LDL' cholesky factorization   which we are using
 in Matrix in other places anyway.
 
 Thanks again for your help!
 Martin Maechler, ETH Zurich
 
 Ravi 
 --
 Ravi Ravi Varadhan, Ph.D.
 Ravi Assistant Professor,  The Center on Aging and Health
 Ravi Division of Geriatric Medicine and Gerontology
 Ravi Johns Hopkins University
 Ravi Ph: (410) 502-2619
 Ravi Fax: (410) 614-9625
 Ravi Email:  [EMAIL PROTECTED]
 Ravi Webpage:
 http://www.jhsph.edu/agingandhealth/People/Faculty/Varadhan.html
 Ravi 
 --
 
  -Original Message-
  From: [EMAIL PROTECTED] [mailto:r-help-
  [EMAIL PROTECTED] On Behalf Of Duncan Murdoch
  Sent: Friday, July 21, 2006 9:45 AM
  To: roger bos
  Cc: RHelp
  Subject: Re: [R] positive semi-definite matrix
 
  On 7/21/2006 8:59 AM, roger bos wrote:
   I have a covariance matrix that is not positive semi-definite
 matrix and
  I
   need it to be via some sort of adjustment.  Is there any R
 routine or
   package to help me do this?
 
  I think you need to be more specific about what have and what you
 want,
  but if the matrix is symmetric and nearly positive semi-definite
 (but
  not exactly because of rounding error), you might try something
 like
 
  fixit - function(A) {
  eig - eigen(A, symmetric = TRUE)
  eig$values - pmax(0, eig$values)
  return(eig$vectors %*% diag(eig$values) %*% t(eig$vectors))
  }
 
  Rounding error means this is not guaranteed to be positive
  semi-definite, but it will be very close.
 
  Duncan Murdoch


__
R-help@stat.math.ethz.ch mailing list
https://stat.ethz.ch/mailman/listinfo/r-help
PLEASE do read the posting guide http://www.R-project.org/posting-guide.html
and provide commented, minimal, self-contained, reproducible code.


[R] multicomp plotting

2006-07-21 Thread Nair, Murlidharan T

I am using the multcomp package for doing multiple comparisons. Since
the data I am handling is huge the number of comparisons are also large.
I am interested in:
1  Breaking down my plots to get rid of the clutter that happens when
plotting the entire data set. How do I pass only part of the data to the
plot function ?
 
fungus.cirec-simint(Fungus.yield~Habitat,data=fungus,conf.level=0.95,ty
pe =c(Tukey))
plot(fungus.cirec)  #This plots the entire data. I want to plot part of
the data only
 
2I am also interested in getting rid of the field name associated with
each categorical variable. 
Here is what the part of the data looks like
 
Habitat Fungus.yield
Birch 20.83829053
Birch 22.9718181
Birch 22.28216829
Birch 24.23136797
Birch 22.32147961
Birch 20.30783598
Oak 27.24047258
Oak 29.7730014
Oak 30.12608508
Oak 25.76088669
Oak 30.14750974
Hornbeam 17.05307949
Hornbeam 15.32805111
Hornbeam 18.26920177
Hornbeam 21.30987049
Hornbeam 21.7173223


It plots labels as HabitatBirch-HabitatOak for example.  How do I get
rid of the field name Habitat in the plot? 
 
3 How do I tell the method to mark the significant comparisons? (i.e
those that do not intersect the zero line).
 
Thanks ../Murli

__
R-help@stat.math.ethz.ch mailing list
https://stat.ethz.ch/mailman/listinfo/r-help
PLEASE do read the posting guide http://www.R-project.org/posting-guide.html
and provide commented, minimal, self-contained, reproducible code.


[R] character to vector

2006-07-21 Thread ajkorman
I have an object of mode character that contains a long sequence of letters. 
How can I convert this object into a vector with each element of the vector
containing a single letter?  Essentially, I want to break the long string of
letters into many individual letters.

Thanks for the help.

Alex

__
R-help@stat.math.ethz.ch mailing list
https://stat.ethz.ch/mailman/listinfo/r-help
PLEASE do read the posting guide http://www.R-project.org/posting-guide.html
and provide commented, minimal, self-contained, reproducible code.


[R] table elemets testing

2006-07-21 Thread Emanuele Mazzola
Hi everybody,

i'm dealing with some percentage tables, of which i should test rowwise if 
the entries are sgnificantly equal or not. Namely, on row 1, test H0: 
element 1= element2, H0: element 1= element3...H0: element 2= element3...H0: 
element n-1= element n. The same on the other rows.

Anybody knows how this can be done in quick way? I don't have large 
matrices, but it seems quite boring...

Thank you very much in advance for your answering,
Emanuele

__
R-help@stat.math.ethz.ch mailing list
https://stat.ethz.ch/mailman/listinfo/r-help
PLEASE do read the posting guide http://www.R-project.org/posting-guide.html
and provide commented, minimal, self-contained, reproducible code.


Re: [R] character to vector

2006-07-21 Thread Marc Schwartz (via MN)
On Fri, 2006-07-21 at 15:15 -0500, [EMAIL PROTECTED] wrote:
 I have an object of mode character that contains a long sequence of letters. 
 How can I convert this object into a vector with each element of the vector
 containing a single letter?  Essentially, I want to break the long string of
 letters into many individual letters.
 
 Thanks for the help.
 
 Alex

 letters
 [1] a b c d e f g h i j k l m n o p q
[18] r s t u v w x y z

 MyChar - paste(letters, collapse = )

 MyChar
[1] abcdefghijklmnopqrstuvwxyz

 MyVec - unlist(strsplit(MyChar, ))

 MyVec
 [1] a b c d e f g h i j k l m n o p q
[18] r s t u v w x y z

See ?strsplit and ?unlist and of course, ?paste for the reverse
operation as above.

HTH,

Marc Schwartz

__
R-help@stat.math.ethz.ch mailing list
https://stat.ethz.ch/mailman/listinfo/r-help
PLEASE do read the posting guide http://www.R-project.org/posting-guide.html
and provide commented, minimal, self-contained, reproducible code.


[R] One arm survival sample calculations

2006-07-21 Thread Hamilton, Cody

Does anyone know of a function that computes the necessary sample size
to reject a null median survival in favor of a given alternative median
survival in a one arm trial?



Cody Hamilton, Ph.D

Institute for Health Care Research and Improvement

Baylor Health Care System

(214) 265-3618





This e-mail, facsimile, or letter and any files or attachments transmitted with 
it contains information that is confidential and privileged. This information 
is intended only for the use of the individual(s) and entity(ies) to whom it is 
addressed. If you are the intended recipient, further disclosures are 
prohibited without proper authorization. If you are not the intended recipient, 
any disclosure, copying, printing, or use of this information is strictly 
prohibited and possibly a violation of federal or state law and regulations. If 
you have received this information in error, please notify Baylor Health Care 
System immediately at 1-866-402-1661 or via e-mail at [EMAIL PROTECTED] Baylor 
Health Care System, its subsidiaries, and affiliates hereby claim all 
applicable privileges related to this information.
[[alternative HTML version deleted]]

__
R-help@stat.math.ethz.ch mailing list
https://stat.ethz.ch/mailman/listinfo/r-help
PLEASE do read the posting guide http://www.R-project.org/posting-guide.html
and provide commented, minimal, self-contained, reproducible code.


Re: [R] Weibull distribution

2006-07-21 Thread Leaf Sun
Thanks for the suggestion! I switched to optimize(), al - optimize(f.fn, lower 
= 0.1, upper =100,tol=0.001);
the warnings were gone and it works stably. 
But when I tried  al - uniroot(f.fn, lower = 0.1, upper =100,tol=0.001);
error occured: f() values at end points not of opposite sign. The error seems 
to me like there is no root found within the interval. I was not able to solve 
this problem.

Thanks!

Leaf





- Original Message -

From: Thomas Lumley,  [EMAIL PROTECTED]
Sent: 2006-07-21,  09:35:11
To: Valentin Dimitrov, [EMAIL PROTECTED]
Subject:  Re: [R] Weibull distribution
  
On  Fri,  21  Jul  2006,  Valentin  Dimitrov  wrote:

  Dear  Leaf,

  I  modified  your  code  as  follows:

  gamma.fun   -  function(mu,sd,start=100)
  {
  f.fn   -  function(alpha)
  {abs(sd^2-mu^2/(gamma(1+1/alpha))^2*(gamma(1+2/alpha)-(gamma(1+1/alpha))^2))}
  alpha   -  optim(start,  f.fn)
  beta   -  mu/gamma(1+1/alpha$par)
  return(list=c(a=alpha$par,b=beta));
  }

  Now  it  works  properly.

  First,  I  added  an  abs().  You  tried  to  solve  an
  equation  by  means  of  the  R-function  optim(),  which
  finds  a  minimum.  That's  why  you  can  find  the  solution
  of  f(x)=a  through  minimization  of  abs(f(x)-a).
  Second,  I  deleted  the  optim-method  BFGS  from  the
  optim()  function,  because  it  is  not  appropriate  in
  this  case.

optim()  is  not  appropriate  at  all  in  this  case  --  its  help  page  
says  to  
use  optimize()  for  one-dimensional  problems.

In  fact,  in  one  dimension  there  isn't  any  need  to  resort  to  
optimization  
when  you  really  want  root-finding,  and  uniroot()  is  more  appropriate  
than  
optimize().


  -thomas

[[alternative HTML version deleted]]

__
R-help@stat.math.ethz.ch mailing list
https://stat.ethz.ch/mailman/listinfo/r-help
PLEASE do read the posting guide http://www.R-project.org/posting-guide.html
and provide commented, minimal, self-contained, reproducible code.


Re: [R] optim()

2006-07-21 Thread Iris Zhao
Dear Spencer,

Thank you very much for your helpful reply. I was trying to reproduce a
table in one paper. After I modified my code according to your suggestion, I
was able to get the results that are very close to those in the paper. It
seems the starting values of the parameters to be optimized are very
crutial. So I will have different optimal values for different starting
vectors. How can I be sure the min value returned by optim() is the true
optimal value?

I am also curious why you choose the constant penalty to handle the
constraint in the first place. Why not use lagrange multiplier method to
eliminate the constraint?

Thanks again. I am grateful for your help.

Best regards,
Iris


On 7/18/06, Spencer Graves [EMAIL PROTECTED] wrote:

  I had good luck translating constrained into unconstrained
 problems
 and then optimizing the unconstrained problem.  Have you tried something
 like the following:

 Define:
  z = c(z1, z2, z3), where p1=1/(1+exp(-z1), etc.  This translates
 the
 constraints on the p's to

  G(z) = P*(f1(z)-r12*f2(z))^2-f1(z)

 where f1(z) = f1(p1(z1), p2(z2), p3(z3), and similarly for f2(z), and
 where P = a penalty term,
 and r12 = (1-c)*k1/(c*(1-k1).

  Can f2(z) ever go outside (0, 1)?  If yes, I would modify G(z) by
 adding a term like (min(0, f2(z), 1-f2(z))^2)

  If I haven't made a math error, your problem should translate
 into
 this form.  I first solve this problem for z with P small like 1.  Then
 after I've got a solution for that, I increase P to 2, then 10, then
 100, etc., until the penalty is so great that the desired equality has
 been effectively achieved.

  With 'P' fixed, 'optim' should handle this kind of problem
 handily.
 To learn how, I suggest you work through the examples in the ?optim help
 page.  I'd ignore the gradient, at least initially.  A silly math error
 in computing the gradient can delay a solutions unnecessarily.  If you
 need to solve thousands of problems like this for  different values of
 k1 and 'c', I might later program the gradient.  However, I would not do
 that initially.

  Also, if you are not already familiar with Venables and Ripley
 (2002)
 Modern Applied Statistics with S, 4th ed. (Springer -- or an earlier
 edition), I would encourage you to spend some quality time with this
 book.  It can help you with 'optim', with contour plots, etc.

  Hope this helps,
  Spencer Graves

 Iris Zhao wrote:
  Dear all,
 
 
 
  I am working on optimization problem and have some trouble running
 optim().
  I have two functions (f1, f2) and 4 unknown parameters (p1, p2, p3, p4).
  Both f1 and f2 are functions of p1, p2, and p3, denoted by f1(p1, p2,
 p3)
  and f2(p1,p2,p3) respectively.
 
 
 
  The goal is to maximize f1(p1, p2, p3) subject to two constraints:
 
  (1)  c = k1*p4/(k1*p4+(1-k1)*f1(p1,p2,p3)), where c and k1 are some
 known
  constants
 
  (2)  p4 = f2(p1, p2, p3)
 
  In addition, each parameter ranges from 0 to 1, and both f1 and f2
 involve
  integrations.
 
 
 
  I tried to use lagrange multipliers to eliminate two equality
 constraints
  and then use optim() to find the maximum value and optimal parameter
  estimates.
 
  So I let fn be f1+lambda1*(c- k1*p4/(k1*p4+(1-k1)*f1(p1,p2,p3))) +
  lambda2(p4-f2(p1,p2,p3)). The error message I got was Error in fn(par,
 ...)
  : recursive default argument reference.
 
 
 
  I wonder whether current build-in functions in R can do this type of
 jobs.
  Any suggestion will be greatly appreciated.
 
 
 
  Iris
 
[[alternative HTML version deleted]]
 
  __
  R-help@stat.math.ethz.ch mailing list
  https://stat.ethz.ch/mailman/listinfo/r-help
  PLEASE do read the posting guide!
 http://www.R-project.org/posting-guide.html


[[alternative HTML version deleted]]

__
R-help@stat.math.ethz.ch mailing list
https://stat.ethz.ch/mailman/listinfo/r-help
PLEASE do read the posting guide http://www.R-project.org/posting-guide.html
and provide commented, minimal, self-contained, reproducible code.


Re: [R] connection to X11 problem

2006-07-21 Thread Ted Harding
On 21-Jul-06 Paquet, Agnes wrote:
 Dear List,
 
 I am a new Mac user and I am having problem generating png (or jpeg)
 using the GUI version of R. I installed R-2.3.1.dmg (custom install
 with
 everything selected) and X11User.pkg but I am still getting the
 following X11 connection error when I try to generate a png (or a
 jpeg):
 
 Error in X11(paste(png::, filename, sep = ), width, height,
 pointsize,  : 
 unable to start device PNG
 In addition: Warning message:
 unable to open connection to X11 display ''
 
 I tried to set up the DISPLAY variable using the command:
 
 Sys.putenv(DISPLAY=:0)
 
 but I am still running into the same problem. 

Like Marc, I don't use a Mac either. But the underlying BSD OS
is basically similar to Linux. On Linux, my primary X11 DISPLAY
envvar would be :0.0, so (at a guess) I suggest you try

  Sys.putenv(DISPLAY=:0.0)

Hoping this helps!
Ted.


E-Mail: (Ted Harding) [EMAIL PROTECTED]
Fax-to-email: +44 (0)870 094 0861
Date: 21-Jul-06   Time: 23:34:05
-- XFMail --

__
R-help@stat.math.ethz.ch mailing list
https://stat.ethz.ch/mailman/listinfo/r-help
PLEASE do read the posting guide http://www.R-project.org/posting-guide.html
and provide commented, minimal, self-contained, reproducible code.


Re: [R] Parameterization puzzle

2006-07-21 Thread Murray Jorgensen
Thanks to Brian and Berwin with there help. I faced a double problem in 
that I not only wanted to fit the model but I also wanted to do so in 
such a way that it would seem natural for a classroom example.

The moral seems to be that I should do the orthogonal polynomial stuff 
outside the model formula. Here then is my solution:

pyears - scan()
18793 52407 10673 43248 5710 28612 2585 12663 1462 5317

deaths - scan()
2 32 12 104 28 206 28 186 31 102

l - log(pyears)
Smoke - gl(2,1,10,labels=c(No,Yes))
Age - gl(5,2,10,labels=c(35-44,45-54,55-64,65-74,75-84),
ordered=TRUE)
mod1.glm - glm(deaths ~ Age * Smoke + offset(l),family=poisson)
summary(mod1.glm)
age - as.numeric(Age)
age1 - poly(age,2)[,1]
age2 - poly(age,2)[,2]
mod2.glm - aso1.glm - glm(deaths ~ age1 + age2 + Smoke +
   age1:Smoke + offset(l),family=poisson)
summary(mod2.glm)

The final summary then comes out looking like this:

   Estimate Std. Error z value Pr(|z|)
(Intercept)-5.8368 0.1213 -48.103   2e-16 ***
age15.3238 0.4129  12.893   2e-16 ***
age2   -1.0460 0.1448  -7.223 5.08e-13 ***
SmokeYes0.5183 0.1262   4.106 4.02e-05 ***
age1:SmokeYes  -1.3755 0.4340  -3.169  0.00153 **


which is just what I wanted.

Cheers,  Murray Jorgensen

Prof Brian D Ripley wrote:
 On Fri, 21 Jul 2006, Berwin A Turlach wrote:
 
 G'day all,

 BDR == Prof Brian Ripley [EMAIL PROTECTED] writes:

BDR R does not know that poly(age,2) and poly(age,1) are linearly
BDR dependent.
 Indeed, I also thought that this is the reason of the problem.

BDR (And indeed they only are for some functions 'poly'.)
 I am surprised about this.  Should probably read the help page of
 'poly' once more and more carefully.
 
 My point was perhaps simpler: if you or I or Murray had a function 
 poly() in our workspace, that one would be found, I think.  (I have not 
 checked the ramifications of namespaces here, but I believe that would 
 be the intention, that formulae are evaluated in their defining 
 environment.)  So omly when the model matrix is set up could the linear 
 dependence be known (and there is nothing in the system poly() to tell 
 model.matrix).
 
 What is sometimes called extrinsic aliasing is left to the fitting 
 function, which seems to be to do a sensible job provided the main 
 effect is in the model.  Indeed, including interactions without main 
 effects (as Murray did) often makes the results hard to interpret.
 
BDR I cannot reproduce your example ('l' is missing), [...]
 My guess is that 'l' is 'pyears'.  At least, I worked under that
 assumption.
 
 Apparently l = log(pyears), which was my uncertain guess.
 
 Interestingly, on my machine (using R 2.3.1, 2.2.1 and 2.1.1) I cannot
 fit any of the Poisson GLM that Murray tried.  I always get the error
 message:

 Error: no valid set of coefficients has been found: please supply 
 starting values
 
 Related to the offset, I believe.
 

 But I have to investigate this further.  I can fit binomial models
 that give me similar answers.

BDR [...] but perhaps
BDR glm(deaths ~ poly(age,2) + poly(age,1)*Smoke + offset(l),
BDR poisson)
BDR was your intention?
 In this parameterisation a 'poly(age,1)' term will appear among the
 coefficients with an estimated value of NA since it is aliased with
 'poly(age, 2)1'.  So I don't believe that this was Murray's intention.

 The only suggestion I can come up with is:

 summary(glm(cbind(deaths, l-deaths) ~ age*Smoke+I(age^2), 
 family=binomial))

 [...]

 Coefficients:
  Estimate Std. Error z value Pr(|z|)
 (Intercept)  -10.798950.45149 -23.918   2e-16 ***
 age2.378920.20877  11.395   2e-16 ***
 SmokeYes   1.445730.37347   3.871 0.000108 ***
 I(age^2)  -0.197060.02749  -7.168  7.6e-13 ***
 age:SmokeYes  -0.308500.09756  -3.162 0.001566 **

 [...]

 Which doesn't use orthogonal polynomials anymore.  But I don't see how
 you can fit the model that Murray want to fit using orthogonal
 polynomials given the way R's model language operates.

 So I guess the Poisson GLM that Murray wants to fit is:

glm(deaths~ age*Smoke+I(age^2)+offset(l), family=poisson)

 Cheers,

Berwin

 == Full address 
 Berwin A Turlach  Tel.: +61 (8) 6488 3338 (secr)
 School of Mathematics and Statistics+61 (8) 6488 3383 (self)
 The University of Western Australia   FAX : +61 (8) 6488 1028
 35 Stirling Highway
 Crawley WA 6009e-mail: [EMAIL PROTECTED]
 Australiahttp://www.maths.uwa.edu.au/~berwin


 

-- 
Dr Murray Jorgensen  http://www.stats.waikato.ac.nz/Staff/maj.html
Department of Statistics, University of Waikato, Hamilton, New Zealand
Email: [EMAIL PROTECTED]Fax 7 838 4155
Phone  +64 7 838 4773 wkHome +64 7 825 0441Mobile 021 1395 862


Re: [R] connection to X11 problem: problem fixed

2006-07-21 Thread Paquet, Agnes
Hi,

I finally managed to make it work (I just needed to have a X11 window
open). 

Thank you very much for your help.

Agnes

-Original Message-
From: Ted Harding [mailto:[EMAIL PROTECTED] 
Sent: Friday, July 21, 2006 3:34 PM
To: Paquet, Agnes
Cc: r-help@stat.math.ethz.ch
Subject: RE: [R] connection to X11 problem

On 21-Jul-06 Paquet, Agnes wrote:
 Dear List,
 
 I am a new Mac user and I am having problem generating png (or jpeg)
 using the GUI version of R. I installed R-2.3.1.dmg (custom install
 with
 everything selected) and X11User.pkg but I am still getting the
 following X11 connection error when I try to generate a png (or a
 jpeg):
 
 Error in X11(paste(png::, filename, sep = ), width, height,
 pointsize,  : 
 unable to start device PNG
 In addition: Warning message:
 unable to open connection to X11 display ''
 
 I tried to set up the DISPLAY variable using the command:
 
 Sys.putenv(DISPLAY=:0)
 
 but I am still running into the same problem. 

Like Marc, I don't use a Mac either. But the underlying BSD OS
is basically similar to Linux. On Linux, my primary X11 DISPLAY
envvar would be :0.0, so (at a guess) I suggest you try

  Sys.putenv(DISPLAY=:0.0)

Hoping this helps!
Ted.


E-Mail: (Ted Harding) [EMAIL PROTECTED]
Fax-to-email: +44 (0)870 094 0861
Date: 21-Jul-06   Time: 23:34:05
-- XFMail --

__
R-help@stat.math.ethz.ch mailing list
https://stat.ethz.ch/mailman/listinfo/r-help
PLEASE do read the posting guide http://www.R-project.org/posting-guide.html
and provide commented, minimal, self-contained, reproducible code.


Re: [R] unexpected results

2006-07-21 Thread nathan

Thanks,

You are right about it being a tab delimited file - I should have spotted
that.

I am now getting an error saying that line 4 did not have 27 elements but
will fiddle around and try to work it out - I'm guessing because I have some
empty feild its causing problems. 

Anyway thanks for the differnt bits of help
-- 
View this message in context: 
http://www.nabble.com/unexpected-results-tf1979786.html#a5441821
Sent from the R help forum at Nabble.com.

__
R-help@stat.math.ethz.ch mailing list
https://stat.ethz.ch/mailman/listinfo/r-help
PLEASE do read the posting guide http://www.R-project.org/posting-guide.html
and provide commented, minimal, self-contained, reproducible code.


[R] Putting x into the right subset

2006-07-21 Thread John Wiedenhoeft
Dear all,

I'm sorry I have to bother you with this newbie stuff.

Within a loop I obtain pairs of values x - c(a, b). An empty set M is
defined before the loop (as a list or whatever). Now I want to do the
following: if there is a vector y in M  that contains at least one of
the values of x, then x shall be concatenated to y. If y doesn't contain
any of the values in x, then x shall be put into M as a new vector. I
first imagined it to be trivial but I'm having a hard time with the
if-statement. I tried to define a function contains(value, vector),
which returns FALSE or TRUE, but R complains that I tried to execute a
non-function...

Could anybody help a despaired newbie, please!

Thanks a lot,
John

__
R-help@stat.math.ethz.ch mailing list
https://stat.ethz.ch/mailman/listinfo/r-help
PLEASE do read the posting guide http://www.R-project.org/posting-guide.html
and provide commented, minimal, self-contained, reproducible code.


Re: [R] Weibull distribution

2006-07-21 Thread Valentin Dimitrov
It seems to me that not for all values of mu and sd
there is a Weibull distribution with mean=mu and
variance=sd^2.
the programm with optimize(f.fn) finds always a
solution, but this is not necessarily what we need,
because the minimum of (abs(f(x)) is not always 0.
Suppose f(x)=2+x^2, then optimize(x) finds x=0, but
x=0 is not a root of f(x)=0.
That's why I agree with Thomas Lumley, that uniroot()
could be more appropriate than optim and optimize.

Best regards,
Valentin

--- Leaf Sun [EMAIL PROTECTED] wrote:

 Thanks for the suggestion! I switched to optimize(),
 al - optimize(f.fn, lower = 0.1, upper
 =100,tol=0.001);
 the warnings were gone and it works stably. 
 But when I tried  al - uniroot(f.fn, lower = 0.1,
 upper =100,tol=0.001);
 error occured: f() values at end points not of
 opposite sign. The error seems to me like there is
 no root found within the interval. I was not able to
 solve this problem.
 
 Thanks!
 
 Leaf
 
 
 
 
 
 - Original Message -
 
 From: Thomas Lumley,  [EMAIL PROTECTED]
 Sent: 2006-07-21,  09:35:11
 To: Valentin Dimitrov, [EMAIL PROTECTED]
 Subject:  Re: [R] Weibull distribution
   
 On  Fri,  21  Jul  2006,  Valentin  Dimitrov  wrote:
 
   Dear  Leaf,
 
   I  modified  your  code  as  follows:
 
   gamma.fun   -  function(mu,sd,start=100)
   {
   f.fn   -  function(alpha)
  

{abs(sd^2-mu^2/(gamma(1+1/alpha))^2*(gamma(1+2/alpha)-(gamma(1+1/alpha))^2))}
   alpha   -  optim(start,  f.fn)
   beta   -  mu/gamma(1+1/alpha$par)
   return(list=c(a=alpha$par,b=beta));
   }
 
   Now  it  works  properly.
 
   First,  I  added  an  abs().  You  tried  to 
 solve  an
   equation  by  means  of  the  R-function 
 optim(),  which
   finds  a  minimum.  That's  why  you  can  find 
 the  solution
   of  f(x)=a  through  minimization  of 
 abs(f(x)-a).
   Second,  I  deleted  the  optim-method  BFGS 
 from  the
   optim()  function,  because  it  is  not 
 appropriate  in
   this  case.
 
 optim()  is  not  appropriate  at  all  in  this 
 case  --  its  help  page  says  to  
 use  optimize()  for  one-dimensional  problems.
 
 In  fact,  in  one  dimension  there  isn't  any 
 need  to  resort  to  optimization  
 when  you  really  want  root-finding,  and 
 uniroot()  is  more  appropriate  than  
 optimize().
 
 
   -thomas


__
R-help@stat.math.ethz.ch mailing list
https://stat.ethz.ch/mailman/listinfo/r-help
PLEASE do read the posting guide http://www.R-project.org/posting-guide.html
and provide commented, minimal, self-contained, reproducible code.


[R] nested repeated measures in R

2006-07-21 Thread Eric C Merten
R help,

How would I input data, verify assumptions, and run a nested repeated
measures ANOVA using R?  I have STATION nested in SITE nested in BLOCK with
measurements repeated for five YEARs.  All are random variables and it's only
slightly unbalanced.  I'm trying to characterize spatiotemporal variation in
stream habitat variables.  Thanks for your help!

Eric Merten

__
R-help@stat.math.ethz.ch mailing list
https://stat.ethz.ch/mailman/listinfo/r-help
PLEASE do read the posting guide http://www.R-project.org/posting-guide.html
and provide commented, minimal, self-contained, reproducible code.


Re: [R] optim()

2006-07-21 Thread Spencer Graves
  1.  I'd be worried any time the answer depended very much on the 
starting values.  It suggests that the objective function is not well 
behaved, and I must be very careful to make sure I get an appropriate 
answer, and I'm not being misled by round-off, etc.

  2.  I would NOT use a constant penalty;  I'd start with a small 
constant penalty, then increase it gradually until I had a solution that 
honestly satisfied the constraints.

  3.  Alternatively, you could use Lagrange multipliers by redefining 
your objective function and including the Lagrange multiplier(s) as 
other paremeter(s) to be estimated.  That sounds like a sensible idea, 
but I have no experience trying that.  I would expect it might work fine 
provided the objective function with constraints were all differentiable 
and sufficiently smooth to admit only one optimum.

  4.  However, I believe you will gain more insight into the problem by 
trying to reduce the number of unknowns rather than increase them.  For 
example, I noted in my earlier reply that your constraints are 
equivalent to f1 = r12*f2, with r12 = r12 = (1-c)*k1/(c*(1-k1) = a 
constant determined from known constants in your previous problem 
statement.  If it were my problem, I might focus on trying to use this 
constraint to determine one of (p1, p2, p3) as a function of the other 
two.  For example, for what combinations in the (p1, p2) space is there 
a single, unique solution, when is there 0 and when are there 2, 3, or 
more?

  To accomplish that, I might use 'expand.grid' to generate many 
different combinations of (p1, p2) and then use 'optim' to minimize SSD 
= (f1-r12*f2)^2 over variations in p3.  (Of course, if it were easier to 
solve for p1 or p2 in terms of the other two, I might do that.)  For 
each combination of (p1, p2), I'd store the resulting values of SSD, p3, 
and f1.  Then if any of the SSD values were numerically greater than 0, 
I'd worry about those cases.  Otherwise, I'd look at contour and 
perspective plots of p3 and f1 vs. p1 and p2 to try to generate some 
insight into this problem -- and perhaps generate a simple way to solve 
for p3 and f1 from p1 and p2.

  Make sense?
  Hope this helps.
  Spencer Graves

Iris Zhao wrote:
 Dear Spencer,
 
 Thank you very much for your helpful reply. I was trying to reproduce a
 table in one paper. After I modified my code according to your suggestion, I
 was able to get the results that are very close to those in the paper. It
 seems the starting values of the parameters to be optimized are very
 crutial. So I will have different optimal values for different starting
 vectors. How can I be sure the min value returned by optim() is the true
 optimal value?
 
 I am also curious why you choose the constant penalty to handle the
 constraint in the first place. Why not use lagrange multiplier method to
 eliminate the constraint?
 
 Thanks again. I am grateful for your help.
 
 Best regards,
 Iris
 
 
 On 7/18/06, Spencer Graves [EMAIL PROTECTED] wrote:
  I had good luck translating constrained into unconstrained
 problems
 and then optimizing the unconstrained problem.  Have you tried something
 like the following:

 Define:
  z = c(z1, z2, z3), where p1=1/(1+exp(-z1), etc.  This translates
 the
 constraints on the p's to

  G(z) = P*(f1(z)-r12*f2(z))^2-f1(z)

 where f1(z) = f1(p1(z1), p2(z2), p3(z3), and similarly for f2(z), and
 where P = a penalty term,
 and r12 = (1-c)*k1/(c*(1-k1).

  Can f2(z) ever go outside (0, 1)?  If yes, I would modify G(z) by
 adding a term like (min(0, f2(z), 1-f2(z))^2)

  If I haven't made a math error, your problem should translate
 into
 this form.  I first solve this problem for z with P small like 1.  Then
 after I've got a solution for that, I increase P to 2, then 10, then
 100, etc., until the penalty is so great that the desired equality has
 been effectively achieved.

  With 'P' fixed, 'optim' should handle this kind of problem
 handily.
 To learn how, I suggest you work through the examples in the ?optim help
 page.  I'd ignore the gradient, at least initially.  A silly math error
 in computing the gradient can delay a solutions unnecessarily.  If you
 need to solve thousands of problems like this for  different values of
 k1 and 'c', I might later program the gradient.  However, I would not do
 that initially.

  Also, if you are not already familiar with Venables and Ripley
 (2002)
 Modern Applied Statistics with S, 4th ed. (Springer -- or an earlier
 edition), I would encourage you to spend some quality time with this
 book.  It can help you with 'optim', with contour plots, etc.

  Hope this helps,
  Spencer Graves

 Iris Zhao wrote:
 Dear all,



 I am working on optimization problem and have some trouble running
 optim().
 I have two functions (f1, f2) and 4 unknown parameters (p1, p2, p3, p4).
 Both f1 and f2 are functions